You are on page 1of 51

Graber and Wilbur's Family Medicine

Examination and Board Review, Fifth


Edition Brigit Ray
Visit to download the full and correct content document:
https://ebookmass.com/product/graber-and-wilburs-family-medicine-examination-and-
board-review-fifth-edition-brigit-ray/
Graber and Wilbur's
Family Medicine
Examination and Board Review
5TH EDITION

• Covers every topic on the Family


Medicine Boards

• Great for USMLE® Step 3 review

• Features hundreds of clinical cases


covering all specialty areas

• Includes Quick Quizzes, Clinical Pearls,


and Learning Objectives

MARK A. GRABER
BRIGIT E. RAY
JASON K. WILBUR
Graber and Wilbur’s
FAMILY MEDICINE
EXAMINATION
& BOARD REVIEW

FM.indd 1 29-11-2019 18:41:49


Graber and Wilbur’s
FAMILY MEDICINE
EXAMINATION
& BOARD REVIEW
FIFTH EDITION
Editors

Mark A. Graber, MD, MSHCE, FACEP


Clinical Professor
Departments of Family Medicine and Emergency Medicine
Roy J. and Lucille A. Carver College of Medicine
University of Iowa
Iowa City, Iowa

Brigit E. Ray, MD, MME


Clinical Assistant Professor
Department of Family Medicine
Roy J. and Lucille A. Carver College of Medicine
University of Iowa
Iowa City, Iowa

Jason K. Wilbur, MD, FAAFP


Clinical Professor
Department of Family Medicine
Roy J. and Lucille A. Carver College of Medicine
University of Iowa
Iowa City, Iowa

New York/Chicago/San Francisco/Athens/London/Madrid/Mexico City/


New Delhi/Milan/Singapore/Sydney/Toronto

FM.indd 3 29-11-2019 18:41:52


Copyright © 2020 by McGraw-Hill Education. All rights reserved. Except as permitted under the United States Copyright Act of 1976,
no part of this publication may be reproduced or distributed in any form or by any means, or stored in a database or retrieval system,
without the prior written permission of the publisher.

ISBN: 978-1-26-044108-6
MHID: 1-26-044108-3

The material in this eBook also appears in the print version of this title: ISBN: 978-1-26-044107-9,
MHID: 1-26-044107-5.

eBook conversion by codeMantra


Version 1.0

All trademarks are trademarks of their respective owners. Rather than put a trademark symbol after every occurrence of a trademarked
name, we use names in an editorial fashion only, and to the benefit of the trademark owner, with no intention of infringement of the
trademark. Where such designations appear in this book, they have been printed with initial caps.

McGraw-Hill Education eBooks are available at special quantity discounts to use as premiums and sales promotions or for use in corpo-
rate training programs. To contact a representative, please visit the Contact Us page at www.mhprofessional.com.

Notice
Medicine is an ever-changing science. As new research and clinical experience broaden our knowledge, changes in treatment and drug
therapy are required. The authors and the publisher of this work have checked with sources believed to be reliable in their efforts to
provide information that is complete and generally in accord with the standards accepted at the time of publication. However, in view
of the possibility of human error or changes in medical sciences, neither the authors nor the publisher nor any other party who has been
involved in the preparation or publication of this work warrants that the information contained herein is in every respect accurate or com-
plete, and they disclaim all responsibility for any errors or omissions or for the results obtained from use of the information contained in
this work. Readers are encouraged to confirm the information contained herein with other sources. For example and in particular, readers
are advised to check the product information sheet included in the package of each drug they plan to administer to be certain that the
information contained in this work is accurate and that changes have not been made in the recommended dose or in the contraindications
for administration. This recommendation is of particular importance in connection with new or infrequently used drugs.

TERMS OF USE

This is a copyrighted work and McGraw-Hill Education and its licensors reserve all rights in and to the work. Use of this work is subject
to these terms. Except as permitted under the Copyright Act of 1976 and the right to store and retrieve one copy of the work, you may
not decompile, disassemble, reverse engineer, reproduce, modify, create derivative works based upon, transmit, distribute, disseminate,
sell, publish or sublicense the work or any part of it without McGraw-Hill Education’s prior consent. You may use the work for your
own noncommercial and personal use; any other use of the work is strictly prohibited. Your right to use the work may be terminated if
you fail to comply with these terms.

THE WORK IS PROVIDED “AS IS.” McGRAW-HILL EDUCATION AND ITS LICENSORS MAKE NO GUARANTEES OR WAR-
RANTIES AS TO THE ACCURACY, ADEQUACY OR COMPLETENESS OF OR RESULTS TO BE OBTAINED FROM USING
THE WORK, INCLUDING ANY INFORMATION THAT CAN BE ACCESSED THROUGH THE WORK VIA HYPERLINK OR
OTHERWISE, AND EXPRESSLY DISCLAIM ANY WARRANTY, EXPRESS OR IMPLIED, INCLUDING BUT NOT LIMITED TO
IMPLIED WARRANTIES OF MERCHANTABILITY OR FITNESS FOR A PARTICULAR PURPOSE. McGraw-Hill Education and
its licensors do not warrant or guarantee that the functions contained in the work will meet your requirements or that its operation will
be uninterrupted or error free. Neither McGraw-Hill Education nor its licensors shall be liable to you or anyone else for any inaccuracy,
error or omission, regardless of cause, in the work or for any damages resulting therefrom. McGraw-Hill Education has no responsibility
for the content of any information accessed through the work. Under no circumstances shall McGraw-Hill Education and/or its licensors
be liable for any indirect, incidental, special, punitive, consequential or similar damages that result from the use of or inability to use
the work, even if any of them has been advised of the possibility of such damages. This limitation of liability shall apply to any claim or
cause whatsoever whether such claim or cause arises in contract, tort or otherwise.
To the people who make Family Medicine happen. Jogjakarta:
Dr. Mora Claramita, Dr. Adi Heru Husodo, Dr. Wahyudi Istiono,
and Dr. Fitriana Murriya. Moscow: Professor Dr. Gregorii Efimovich
Roitberg, Dr. Olga Sharkun, Dr. Irina Slastnikova, Dr. Janna Dorosh,
and Professor Timothy O’Connor. Haiphong: Dr. Hùng Nguyễn Văn,
Dr. Linh Nguyễn, and Dr. Nguyễn Thuý Hiếu.
—MAG

To my parents, Bill and Bonnie Ray, for always loving, supporting, and
encouraging me to follow my dreams to become a family physician.
To all of my Family Medicine and education mentors including: Drs.
George Bergus, Rick Dobyns, Jason Wilbur, Bob Tallitsch, Jeff Pettit,
and Marcy Rosenbaum. Your guidance and support along this journey
have shaped me into the family physician educator I have become!
—BER

To my many mentors in Family Medicine, including Drs. John Ely,


Mark Graber, Paul James, Gerald Jogerst, and Barcey Levy; and to Dr.
Kate Thoma and all the Family Medicine program directors and faculty
members who work tirelessly (and mostly thanklessly) to develop the
next generation of excellent family doctors. Keep teaching; keep leading!
—JKW

FM.indd 5 29-11-2019 18:41:52


Contents
Contributors..............................................................................................ix 16 Men’s Health............................................................................... 501
Preface..................................................................................................... xiii 17 Dermatology.............................................................................. 525
A Few Words on Studying and Taking the Board Examination...... xv
18 Neurology.................................................................................... 551
1 Emergency Medicine................................................................... 1 19 Ophthalmology......................................................................... 585
2 Cardiology......................................................................................43 20 Otolaryngology......................................................................... 607
3 Pulmonology.............................................................................. 105 21 Care of the Older Patient........................................................ 631
4 Allergy and Immunology....................................................... 141 22 Care of the Surgical Patient................................................... 662
5 Nephrology................................................................................. 150 23 Psychiatry.................................................................................... 693
6 Hematology and Oncology................................................... 183 24 Nutrition and Herbal Medicine............................................ 729
7 Gastroenterology...................................................................... 211 25 Substance Use Disorders....................................................... 742
8 Infectious Diseases................................................................... 253 26 Ethics............................................................................................. 763
9 HIV/AIDS...................................................................................... 281 27 End-of-Life Care......................................................................... 774
10 Endocrinology........................................................................... 296 28 Evidence-Based Medicine..................................................... 788
11 Rheumatology........................................................................... 330 29 Patient-Centered Care............................................................. 800
12 Orthopedics and Sports Medicine..................................... 364 30 Final Examination..................................................................... 813
13 Pediatrics..................................................................................... 395
14 Adolescent Medicine............................................................... 437
Index........................................................................................................841
15 Obstetrics and Women’s Health.......................................... 451

vii

FM.indd 7 29-11-2019 18:41:52


Contributors
Emad Abou-Arab, MD, CME Guru V. Bhoojhawon, MBBS, MD, FAAP
Clinical Assistant Professor Clinical Associate Professor
Department of Family Medicine Department of Pediatrics
Roy J. and Lucille A. Carver College of Medicine Roy J. and Lucille A. Carver College of Medicine
University of Iowa University of Iowa
Iowa City, Iowa Iowa City, Iowa
10. Endocrinology 13. Pediatrics

Daniel M. Anderson, DO
Nicholas R. Butler, MD, MBA
Senior Associate Consultant
Clinical Associate Professor
Department of Neurology
Department of Family Medicine
Mayo Clinic Health System Franciscan Healthcare
Roy J. and Lucille A. Carver College of Medicine
La Crosse, Wisconsin
University of Iowa
18. Neurology
Iowa City, Iowa
A. Ben Appenheimer, MD 21. Care of the Older Patient
Clinical Assistant Professor
Department of Internal Medicine Rachel R. Butler, MD
Division of Infectious Diseases Clinical Assistant Professor
Roy J. and Lucille A. Carver College of Medicine Department of Internal Medicine
University of Iowa Division of Pulmonary, Critical Care, and Occupational
Iowa City, Iowa Medicine
8. Infectious Diseases Roy J. and Lucille A. Carver College of Medicine
9. HIV/AIDS University of Iowa
Iowa City, IA
Stacey Appenheimer, MD 3. Pulmonology
Clinical Assistant Professor
Department of Family Medicine
Meghan Connett, MD
Roy J. and Lucille A. Carver College of Medicine
Clinical Assistant Professor
University of Iowa
Department of Family Medicine
Iowa City, Iowa
Roy J. and Lucille A. Carver College of Medicine
14. Adolescent Medicine
University of Iowa
Iowa City, Iowa
Olivia E. Bailey, MD
15. Obstetrics and Women’s Health
Clinical Associate Professor
Department of Emergency Medicine
Roy J. and Lucille A. Carver College of Medicine Dustin Z. DeYoung, MD
University of Iowa Psychiatrist
Iowa City, Iowa Behavioral Health Associates
1. Emergency Medicine University of California, Los Angeles
Los Angeles, California
Maresi Berry-Stoelzle, MD
Department of Family Medicine Austin R. Fox, MD
Roy J. and Lucille A. Carver College of Medicine Department of Ophthalmology and Visual Sciences
University of Iowa University of Iowa
Iowa City, Iowa Iowa City, Iowa
6. Hematology and Oncology 19. Ophthalmology
ix

FM.indd 9 29-11-2019 18:41:52


x CONTRIBUTORS

Mark A. Graber, MD, MSHCE, FACEP Aaron R. Kunz, DO, MA, MME
Clinical Professor Clinical Assistant Professor
Departments of Family Medicine and Emergency Medicine Department of Family Medicine
Roy J. and Lucille A. Carver College of Medicine Roy J. and Lucille A. Carver College of Medicine
University of Iowa University of Iowa
Iowa City, Iowa Iowa City, Iowa
1. Emergency Medicine 16. Men’s Health
2. Cardiology
7. Gastroenterology Victoria Linares, MD
12. Orthopedics and Sports Medicine CAQ Geriatric Medicine
26. Ethics Clinical Assistant Professor
28. Evidence-Based Medicine Department of Primary Care
30. Final Examination Loyola University Medical Center
Maywood, IL
Erin Hayward, MD 21. Care of the Older Patient
Clinical Assistant Professor
Department of Family Medicine Britt L. Marcussen, MD
University of Iowa CAQ Sports Medicine
Iowa City, Iowa Clinical Associate Professor
22. Care of the Surgical Patient Department of Family Medicine
Roy J. and Lucille A. Carver College of Medicine
Priyanka Iyer, MD, MPH University of Iowa
Clinical Associate Professor Iowa City, Iowa
Department of Internal Medicine 12. Orthopedics and Sports Medicine
Division of Immunology
University of Iowa Clinics and Hospitals
Denise A. Martinez, MD
Iowa City, Iowa
Clinical Associate Professor
11. Rheumatology
Associate Dean for Diversity, Equity, and Inclusion
Department of Family Medicine
Monika Jindal, MD
Roy J. and Lucille A. Carver College of Medicine
Instructor, University of Colorado School of Medicine
University of Iowa
Department of Psychiatry
Iowa City, Iowa
Department of Family Medicine
29. Patient-Centered Care
Denver Health Medical Center
Denver, Colorado
23. Psychiatry Patrick J. McCarthy, MD, MME
Assistant Professor
Nicholas H. Kluesner, MD, FACEP Section of Hospital Medicine, Department of Pediatrics
Associate Medical Director Medical College of Wisconsin/Children’s Hospital of
Department of Emergency Medicine Wisconsin
UnityPoint Health – Des Moines Milwaukee, Wisconsin
Des Moines, Iowa 13. Pediatrics
26. Ethics
Sarah L. Miller, MD, FACEP, FAAP
Jason Kruse, DO Clinical Associate Professor
Department of Internal Medicine Department of Emergency Medicine
Broadlawns Medical Center Roy J. and Lucille A. Carver College of Medicine
Des Moines, Iowa University of Iowa
7. Gastroenterology Iowa City, Iowa
1. Emergency Medicine
Bharat Kumar, MD, MME, FACP, RhMSUS
Department of Internal Medicine and Division of Megan H. Noe, MD, MPH, MSCE
Immunology Instructor
Department of Internal Medicine Department of Dermatology
Roy J. and Lucille A. Carver College of Medicine Brigham & Women’s Hospital
University of Iowa Harvard Medical School
Iowa City, Iowa Boston, Massachusetts
11. Rheumatology 17. Dermatology

FM.indd 10 29-11-2019 18:41:52


Contributors xi

Scott R. Owen, MD Melissa L. Swee, MD, MME


Assistant Professor Clinical Assistant Professor
Director of Facial Plastic and Reconstructive Surgery Department of Internal Medicine
Department of Otolaryngology, Head and Neck Surgery Division of Nephrology
University of Iowa Roy J. and Lucille A. Carver College of Medicine
Iowa City, Iowa University of Iowa
20. Otolaryngology Iowa City, Iowa
5. Nephrology
Juan R. Pagan-Ferrer, MD, DABIM, Ger, HPM
Clinical Assistant Professor Teri Thomsen, MD
Department of Internal Medicine Clinical Associate Professor
Roy J. and Lucille A. Carver College of Medicine Department of Neurology
University of Iowa Roy J. and Lucille A. Carver College of Medicine
Iowa City, Iowa University of Iowa
27. End-of-Life Care Iowa City, Iowa
Sanjeev Patil 18. Neurology
Assistant Professor
Department of Rheumatology and Immunology Alka Walter, MBBS, MS
University of Vermont Medical Center Clinical Assistant Professor
Burlington, VT Department of Family Medicine
Roy J. and Lucille A. Carver College of Medicine
Brigit E. Ray, MD, MME University of Iowa
Clinical Assistant Professor Iowa City, Iowa
Department of Family Medicine 22. Care of the Surgical Patient
Roy J. and Lucille A. Carver College of Medicine
University of Iowa Karolyn A. Wanat, MD, FAAD
Iowa City, Iowa Associate Professor
15. Obstetrics and Women’s Health Department of Dermatology
22. Care of the Surgical Patient Medical College of Wisconsin
30. Final Examination Milwaukee, Wisconsin
17. Dermatology
Wendy W. Shen, MD, PhD
Clinical Associate Professor
Department of Family Medicine Jason K. Wilbur, MD, FAAFP
Roy J. and Lucille A. Carver College of Medicine Clinical Professor
University of Iowa Department of Family Medicine
Iowa City, Iowa Roy J. and Lucille A. Carver College of Medicine
4. Allergy and Immunology University of Iowa
Iowa City, Iowa
Tameem A. Shoukih, MD 2. Cardiology
Clinical Assistant Professor 6. Hematology and Oncology
Departments of Emergency Medicine and Pediatrics 16. Men’s Health
Roy J. and Lucille A. Carver College of Medicine 24. Nutrition and Herbal Medicine
University of Iowa 30. Final Examination
Iowa City, Iowa
12. Orthopedics and Sports Medicine Qiang Zhang, MD
Movement Disorder Fellow
Kelly Skelly, MD Department of Neurology and Iowa Neuroscience Institute
Clinical Associate Professor Physician Scientist Training Program
Department of Family Medicine Clinical NeuroScientist Training Program
Roy J. and Lucille A. Carver College of Medicine Roy J. and Lucille A. Carver College of Medicine
University of Iowa University of Iowa
Iowa City, Iowa Iowa City, Iowa
14. Adolescent Medicine 18. Neurology
C. Blake Sullivan, MD
Resident Physician
Otolaryngology – Head and Neck Surgery
University of Iowa
Iowa City, Iowa
20. Otolaryngology

FM.indd 11 29-11-2019 18:41:52


This page intentionally left blank

FM.indd 6 29-11-2019 18:41:52


Preface
Welcome to the fifth edition of Graber & Wilbur’s Family The first edition was published amid a less-than-friendly
Medicine Examination & Board Review book. We hope that you environment, with declining interest in print media and several
are as excited to be here as we are. The whole book has been well-known board review books already on the market. Because
meticulously updated to not only serve you well as a study guide, our book carried a different tone, readers slowly gravitated to
but also to provide you with cutting edge, up-to-date, informa- it, and its market share grew by word-of-mouth. Engaged and
tion pertinent to your practice. For those of you who love your supportive readers play a huge role!
e-readers and tablets, there is a portable, electronic version of In preparation for work on the fifth edition, we saved all of
the book. You can also now find us on “AccessMedicine.” your e-mails and scoured the Internet for reader comments and
What is new? We have added a succinct guide to the rec- reviews. We read and considered all that we could find—which
ommended health maintenances for children and adults, per- amounted to several hundred readers’ ideas. So, you, the reader,
fect for a quick review. We estimate that 30% of the material have helped shape this book. Keep those e-mails coming!
has been changed since our last edition: think of hepatitis C With all of the board review books out there, why should you
diagnosis and treatment, new cholesterol and blood pressure choose our text? There are two crucial differences between this
guidelines, new heart failure guidelines (and drugs), the ever- book and other board review books on the market. First, we
changing anticoagulation guidelines, etc. have written this book not only to help you pass the boards but
What has not changed? Our essential style remains the same. also to broaden your knowledge of family medicine. The major-
The book is divided into 29 chapters based on body system and ity of questions contain a detailed explanation not only of why
elements of patient care, followed by the “Final Examination” an answer is right but also why the other answers are wrong.
(Chapter 30). The thousands of questions in the book are woven In the rapidly changing world of medical knowledge, we have
into cases, which we hope you will find interesting, practical, endeavored to provide you with the most relevant and up-to-
and relevant. To test your acquisition of knowledge, each case date evidence. When the current evidence is controversial and
ends with the learning objectives. To break the monotony of we are not certain what the American Board of Family Medicine
slogging through a study guide, you will find “Quick Quizzes” (ABFM) will do with it, we acknowledge the uncertainty and try
and “Helpful Tips” peppered throughout each chapter. to help you navigate the current evidence.
A dozen years ago when we wrote the first edition, we made We have tried to make this book as broad and as comprehen-
the decision to use the second-person voice in order to engage sive as possible. In addition to its use as a board review book
the reader better and to give the book a conversational appeal. for family medicine, it can be employed as a general review for
We have tried to keep the book from being boring. Yes, we are primary care physicians, physician assistants, and nurse practi-
aware that this is a study aid. But why must studying be an exer- tioners. Students and residents studying for Step 3 of the licens-
cise in tedium and endurance? It should be enjoyable, appli- ing examination should find the book helpful as well. However,
cable to real life and provide a surprise every now and again. no board review book can possibly cover the entire scope of
You will find (sometimes feeble) attempts at humor throughout family medicine. Use these questions as a guide: what areas are
the book. We have noticed that an occasional reader does not your strengths and what do you need to study further? Each
appreciate our sense of humor. As Abraham Lincoln famously answer of the “Final Examination” is referenced in the book
never said, “You can please some of the people all of the time, so you can go back and review any topic that you might have
and then again, some people just won’t think you’re funny.” missed.
We have been impressed with the level of engagement of In this book, the use of eponymous medical terms such as
our readers. Over the years, we have received scores of e-mails Crohn disease and Wilson disease reflects the current American
from readers who have thanked us, corrected us, and some- Medical Association recommendations for these and simi-
times chided us. No matter the intent of the message, the tone lar terms where the possessive form is dropped. In addition,
is almost universally positive—readers are invested in the book there is a general trend toward using fewer eponyms, such as
and want to offer helpful suggestions. Likewise, the comments Wegener, which has been dropped completely. We have made
posted online (not usually the place one goes for affirmation) note of both new and old terms when we have deemed the old
have been mostly approving, constructive, and enthusiastic. term more recognizable.

xiii

FM.indd 13 29-11-2019 18:41:52


xiv PREFACE

We enjoyed writing this book and we hope that you enjoy to the project, and Jason simply owes her dinner every night …
using it. If you have suggestions or complaints (okay, maybe for a year. Jason thanks his boys, Ken and Ted, who offer a great
all of our jokes aren’t politically correct or even funny), do not distraction from work (like learning to drive—yikes!) and find
hesitate to write us at mark-graber@uiowa.edu, jason-wilbur@ it entertaining that their dad is some form of an author. Finally,
uiowa.edu, or brigit-ray@uiowa.edu. We take your comments as with every edition, Jason must acknowledge that the book
seriously as we endeavor to make studying for the board exami- would never get finished without large amounts of coffee; so,
nation more effective and more fun. he thanks everyone involved in the worldwide production of
We acknowledge and thank all our chapter authors who have coffee, from the pickers on the Central American fincas to the
brought their expertise to bear on this project. We also want local baristas. He’s really hoping that we all do something about
to thank the good people at McGraw-Hill who have edited the climate change to at least save the coffee-growing regions of the
book to keep errors to a minimum and created a handsome and world.
readable layout. Brigit thanks her husband, Austin, for being so patient when
Mark would like to thank you, the reader, for buying this she has been cranky and sleep deprived. She is so proud of her
book. Thanks also to his family: Hetty, Rachel, and Abe (as husband’s hard work and dedication in the completion of his
always). But not to the dogs, Nietzsche and “Vash the Stampede.” research, residency, and fellowship. He has been an inspiration
They need to learn to stay either in or out of the house. No and her greatest sidekick, friend, and love, and she can’t wait
more of this back and forth. Music that has kept Mark awake: to start this new chapter in their life together; by FINALLY liv-
“Hellborg, Lane, and Sipe” (check out “Time is the Enemy” and ing in the same location! She would like to acknowledge the
“Personae” [yes, it is spelled with an “e”]), Stephane Wrembel “Academy” (no really, she’s not joking) as she has spent many
(Barbes-Brooklyn is Mark’s favorite but you can’t go wrong), hours on her couch with the movies and the AAFP editing this
and the Kinks. Finally, thanks to his bicycle for keeping Mark book and completing her CME questions. Lastly, she would like
sane … although some would argue this point. to thank the open roads, blue skies, and sunshine for always
Jason thanks his loving and supportive family. After some providing her with much needed “run therapy” and happiness.
initial threats, Deb has granted her patience and understanding

FM.indd 14 29-11-2019 18:41:52


A Few Words on Studying and
Taking the Board Examination
Throughout the book, you will find that we give advice on what you probably already have a sense of areas of strength and weak-
we think is likely to be on the examination. That’s what you’re ness. Make sure you address your weak areas with relatively
paying for, right? However, the fourth edition marked the first more time on them.
time that we offered advice on studying for and taking the Next, know what is on the examination. The percentage of
examination. We thought it worthwhile to keep this section for examination content devoted to various systems is posted on
the fifth edition. We recommend you read this section prior to the ABFM website, and we recommend you review it. The top
diving into the rest of the book. systems tested are usually cardiovascular, respiratory, and mus-
While we acknowledge that some people are simply better culoskeletal systems. If you are weak in any of these areas, be
test takers than others, there is good evidence to show that any- certain to focus your studying on them.
one can improve his or her scores. In fact, examination scores Now, what material should you use when studying? Some of
are directly proportional to time spent studying for the exami- our readers have been overly kind, suggesting in their reviews
nation (although this association grows weaker for those who that this book is the only study tool needed for the board exami-
have high scores already). The point is, you don’t have to be a nation. While we would like to believe it, we cannot endorse this
genius who got a 36 on the ACT in order to rock the ABFM point of view.
Certification/Recertification Examination. But you may need to To get a flavor for the questions on the examination, the
put in the work. best strategy is to go to the source. The ABFM posts its ITE
Your first step in studying for the examination—after pur- for the last 3 years on its website (www.theabfm.org). A login
chasing this book, of course—should be to develop a study is required, which board-certified family physicians should all
plan. Plotting out time and dedicating that time to uninter- have. The ITEs are perhaps the best source for assessing your
rupted study is important. How much time do you have before knowledge—we strongly recommend you use them. Although
the examination? How many hours per week can you devote to we do not recommend relying on the ITEs as your only study aid
studying? When are you most productive in your studying— (obviously; we’re trying to sell books here!), you can use these
morning or night? What are the chances of a worldwide failure as a way to measure your progress as you study. The critiques
of coffee crops? Will a new Star Wars movie open before the are available as well, so you can learn what the ABFM thinks
examination? Thinking through these questions, get a calen- you should know. The ABFM also has extensive information on
dar, mark the examination date, and plot out days and times what you should expect when you sit for the certifying/recerti-
that you will devote to studying. If you have taken the exami- fying examination, including a tutorial that simulates the exam-
nation before and it didn’t turn out so well, you may need to ination. If you are an anxious test-taker, be certain to check out
change your daily work schedule for 2 to 3 months before the the tutorial. While the ABFM has several useful tools, be aware
examination to accommodate studying 10 to 15 hours per that the Self-Assessment Modules are not representative of the
week. We endorse neither “cramming” for the examination nor types of questions you will find on the certifying/recertifying
“adding on” studying to an already full schedule. To get the examination; however, the more recent Continuous Knowledge
most out of studying, you need to approach it like a daily Self-Assessment activity, where the diplomate answers 25 ques-
devotion. tions per quarter, is a closer representation.
In order to maximize your return on your studying and to Another great source for questions is the American Academy
focus on deficiencies, try taking a pre-test. The best pre-test is of Family Physicians (AAFP) website. If you are a member of
the ABFM In-Training Examination (ITE) — keep reading for AAFP, you can access questions for free. They are categorized by
more on this. You can use your results on the pre-test to see body system and can be done in chunks of ten at a time worth
what areas are your weakest. Studying weak areas is less fun 0.25 CME credits. This question bank offers another opportu-
but will net higher yield results than studying areas of relative nity to test your knowledge and determine where you need to
strength. If your practice is narrow in scope (e.g., a hospitalist), focus your studying.

xv

FM.indd 15 29-11-2019 18:41:52


xvi A Few Words on Studying and Taking the Board Examination

Also, the AAFP markets a comprehensive board review self- Read every stem and option carefully. Although we doubt
study course, which will set you back over $1,000 if you are a that the ABFM writes “trick questions,” they do use catch
member and more if you are not. Indeed, it covers everything words/phrases, such as “except,” “most likely,” “first step,” and
you need to know for the examination. But so does this book! “least likely.” If you are not attending to the catch phrase, you are
So, the choice is yours, but we doubt that you will need both our likely to answer the question wrong.
book and the AAFP board examination self-study package. In the past, the ABFM recommended relying on evidence in
What about texts and primary sources? Well, while we would place up to 2 years before the examination rather than the most
admire your perseverance in slogging through whole texts pre- recent medical evidence. Now, the ABFM recommends exam-
paring for the examination, we do not recommend attempt- inees rely on the most up-to-date evidence available. So, when
ing to read cover-to-cover texts like Robert Rakel’s Textbook of you are looking at a question and thinking, “Well, the answer
Family Medicine or reference material like UpToDate. Don’t get last year might have been ‘A’ but now the evidence points to ‘B’.”
us wrong. We like these sources and recommend them to you as Choose “B.”
references as you are studying, but you should not rely on them Successful test-takers do not use grand strategies to outsmart
as your sole study material. Likewise, using primary sources, the question writers; instead they tend to employ a few simple
like medical journals, is impractical as a study foundation but rules when answering multiple-choice questions. These simple
useful to expand your knowledge when you don’t understand rules that follow amount to guidelines that cannot be blindly
something. applied to the entire test, but are often true. No secret to many
As far as board review courses: to each his or her own. If you of you, perhaps, but here they are:
are considering attending a course, the AAFP offers compre-
• Go with your first thoughtful choice unless you have a solid
hensive courses multiple times per year in locations all across
reason to change it (e.g., you misread the question).
the country. For-profit entities provide additional options. If
• Look for catch words in the answers, such as “always” and
you learn best in a live lecture setting, these courses may be a
“never.” These will often be incorrect.
good option for you, but you need not attend a course to get all
that information (c’mon—you’ve got this book!). • Avoid answers with unfamiliar terms (e.g., obscure disease
There are some important basic things you need to know names or rarely performed procedures). These are often
about the examination. As of the writing of this book, the exam- incorrect.
ination is composed of 4 sections, each consisting of 80 multiple • The most detailed answer is often the correct answer.
choice questions and 100 minutes in length. Sections 1, 3 and • If two answers are similar, they are probably both wrong.
4 have questions from a wide variety of family medicine topics. • Stick with family medicine principles (e.g., answers with
Section 2 consists of 40 questions from a chosen module and “more history” or “shared decision making” are more likely
40 questions pertaining to the general breadth of family med- to be correct).
icine. It is best to choose modules with which you are more • If you don’t know, guess and move on. Do not waste time
familiar. For example, if you practice primarily in an emergency deliberating on a single question.
department, you may want to choose Emergent/Urgent Care or
Hospital Medicine rather than Maternity Care (unless you’re Finally, we part offering advice that we know busy doc-
looking for the additional challenge). We highly recommend tors seldom follow: get plenty of rest. Seriously! Be prepared
to check out the ABFM website (https://www.theabfm.org/con- for the examination day by getting a good night’s sleep. Don’t
tinue-certification/cognitive-expertise/one-day-fmc-exam) for stop taking care of your health prior to the examination, and
exam information as this may change. that includes rest. Eat a good breakfast, bring a snack for your
The examination consists entirely of four-item multiple- breaks, and plan to take yourself out for a nice lunch (but skip
choice questions. You are not penalized for guessing. An unan- the martini—you’ve got an examination to finish). Just like a
swered question will always be wrong; whereas, a guessed mountain climber, wear layers. Some of those test-taking cen-
question has a 25% chance of being right. If you have no idea, ters are freezing; some are boiling. Stay positive, take a deep
go ahead and guess. As a corollary to that rule, never exit the breath and keep moving through it. You will pass this thing!
examination without first completing all the items. You cannot Good luck.
return to answer unmarked items.

FM.indd 16 29-11-2019 18:41:52


Emergency Medicine
Olivia E. Bailey, Sarah L. Miller, and Mark A. Graber
1
believes the ingestion occurred about 50 minutes ago as her
▶▶ CASE 1.1 child told her she found the bottle in the bathroom when she
You get a call from a panicked mother because her 4-year-old woke up from her nap. You contemplate gastrointestinal (GI)
drank a bottle of children’s Tylenol. She found the empty bot- decontamination.
tle in her child’s bed after a nap, and her child had been in bed
for 90 minutes. She thinks there were about 3 ounces of liquid Question 1.1.2 Which of the following statements is true
left in the bottle. She is about 35 minutes from the hospital. about gastric lavage?
She states her child weighs 15 kg. A) Except in extraordinary circumstances it should only be
done in the first hour after an overdose
Question 1.1.1 Your advice to her is: B) Patients who have had gastric lavage have higher incidence
A) Induce vomiting to reduce acetaminophen absorption of pulmonary aspiration than patients who have not
B) Have the child eat to slow absorption and proceed directly to C) Patients who undergo gastric lavage have a higher incidence
the hospital of esophageal perforation
C) Proceed to the hospital D) It can push pill fragments beyond the pylorus
D) Breathe deeply and calm down; the amount of acetamino- E) All of the above are true
phen this child could have ingested is harmless
Answer 1.1.2 The correct answer is “E.” All of the options are
Answer 1.1.1 The correct answer is “C.” Proceed to the hospi- true. Generally, the efficacy of gastric lavage is limited. Outcome
tal. “A” is incorrect for a couple of reasons. After 90 minutes, it is data do not support the use of gastric lavage after the first hour.
not likely that there is significant medication left in the stomach In a particularly severe overdose or in an overdose that is likely
and induced vomiting can lead to aspiration (this is true of liq- to delay gastric emptying (e.g., tricyclic antidepressants), you
uids and pills). “B” is incorrect because you do not want to delay might want to consider lavage, but such circumstances are
definitive treatment. “D” is incorrect as this patient may have unusual. Gastric lavage increases the risk of aspiration, esoph-
ingested up to 3 oz (90 mL) of 160 mg/5 mL solution (total dose ageal perforation, and can push pill fragments beyond the
of 2880 mg or 192 mg/kg). pylorus.

Question 1.1.3 After careful consideration, you decided


HELPFUL TIP: not to lavage. She is now 90 minutes after the ingestion.
Acetaminophen is the most common agent involved Her physical exam is normal other than some dried sticky
in pediatric ED visits for over-the-counter medica- liquid on her face, shirt, and hands that smells like cherry
tion exposures. Reasons for these high exposure rates flavoring. The next best step to take in this patient is to:
include the medication’s reputation as “safe,” its ubiq- A) Check blood acetaminophen levels and refer for hemodialy-
uity in medicine cabinets, errors in dosing, as well as sis if markedly elevated
co-administration of medications that also contain B) Administer 5 g/kg of charcoal with sorbitol
acetaminophen. C) Start treatment with N-acetylcysteine (NAC)
D) Prophylactically treat this patient for seizures using
phenytoin
The patient arrives in your emergency department (ED). E) Observe and measure acetaminophen level at 4 hours after
She is alert with stable vital signs. The mother states she now ingestion
1

Ch01.indd 1 29-11-2019 12:14:48


2 FAMILY MEDICINE EXAMINATION & BOARD REVIEW

Answer 1.1.3 The correct answer is “E.” Giving charcoal is charcoal. “B,” “D,” and “E” are all incorrect. While we do have
likely helpful only within the first hour after ingestion, and antidotes for digoxin and opiates (Digibind, naloxone), char-
even this remains controversial. “A” is incorrect because hemo- coal may still be indicated to reduce absorption within the
dialysis is not indicated for acetaminophen (APAP) overdose first hour.
and measuring levels at 90 minutes will not allow appropri- ▶▶ Objectives: Did you learn to …
ate risk stratification for this child. “C” and “D” are incorrect • Manage a patient with an acute ingestion?
because seizure prophylaxis is not indicated in this patient, and
• Describe the appropriate use of gastric lavage and charcoal
although NAC could be initiated for a known dose of >150
administration?
mg/kg, this child clearly had spilled medication on her skin and
• Identify situations where charcoal may not be indicated?
clothing. In order to determine the risk of APAP-induced hepa-
totoxicity and treatment via the Rumack–Matthew nomogram,
a 4-hour APAP level is necessary. QUICK QUIZ: BIOTERRORISM AND THE ATTACK
OF GODZILLA
HELPFUL TIP:
Although frequently given, single dose activated char- Oh, no. Godzilla is attacking Tokyo. And this time it is with
coal has limited or no effect on outcomes of poisonings. It weapons of mass destruction. Which of the following
reduces absorption by about 30% if given within 1 hour properly describes the isolation requirements of a patient
of ingestion and likely has no benefit after 1 hour. It can with pulmonary anthrax?
also cause vomiting with aspiration. For this reason, it A) No isolation necessary. The patient may be in the same room
has fallen out of favor (we don’t remember the last time with an uninfected patient
we used it in our ED). We are not sure what the correct B) Respiratory isolation only
answer on the test will be. C) Respiratory and contact isolation
D) Negative pressure room (such as with tuberculosis) + ­contact
isolation
HELPFUL TIP:
The correct answer is “A.” Pulmonary anthrax is NOT trans-
Do NOT give activated charcoal to patients with an
mitted person to person. Contact isolation is indicated in those
altered mental status or who are otherwise unable to
with cutaneous anthrax and GI anthrax (where diarrhea may be
protect their airway. To prevent aspiration, do not give
infectious).
charcoal to a patient likely to have a seizure (such as
with tricyclic overdose). Godzilla is not done yet… Which of the following drugs
should be used as prophylaxis against inhaled anthrax,
should exposure to aerosolized spores be documented?
Question 1.1.4 Assuming you are using charcoal, for which A) A first-generation cephalosporin
of these overdoses is charcoal NOT indicated? B) Trimethoprim/sulfamethoxazole
A) Acetaminophen C) Ciprofloxacin
B) Aspirin D) A third-generation cephalosporin
C) Iron
D) Digoxin The correct answer is “C.” Fluoroquinolones are the drugs of
E) Opiates choice when treating those exposed to anthrax. Doxycycline
may also be used. Cephalosporins and TMP/SMX are not active
Answer 1.1.4 The correct answer is “C.” Charcoal will not against anthrax.
bind iron. Charcoal will also not bind Caustics/corrosives,
Heavy metals, Alcohols, Rapid-onset cyanide, Chlorine (or Godzilla, frustrated by his failed anthrax attack, is now
iodine), Other insoluble tablets, Aliphatics (hydrocarbons), spreading smallpox. Which of the following is NOT true
or Laxatives (mnemonic: CHARCOAL). Some of you may about smallpox?
have answered “A.” Theoretically, charcoal could interfere A) Isolation is best done at home if possible
with the action of N-acetylcysteine, the antidote for acet- B) The patient is infectious until he or she becomes afebrile
aminophen ingestion by absorbing it. However, this is more C) All lesions are generally in the same stage of evolution,
of a theoretical concern than an actual one. First, the drugs unlike what is seen in varicella
should be used at different times. Charcoal should be given D) Smallpox immunization causes an encephalitis in 1:300,000
immediately, while N-acetylcysteine is given only after 4-hour of which 25% of cases are fatal
levels are available. Second, the doses of N-acetylcysteine rec-
ommended are quite high, and you can give a higher dose if The correct answer is “B.” The patient is infectious until all
you will be using it with charcoal. Finally, intravenous (IV) lesions crust over. Infectivity has nothing to do with the pres-
N-acetylcysteine is available and is obviously not affected by ence or absence of fever. “A” is true. Isolation is best done at

Ch01.indd 2 29-11-2019 12:14:48


CHAPTER 1 • Emergency Medicine 3

home since this will limit spread (as those in the household have Answer 1.2.2 The correct answer is “E.” All of the above find-
likely already been exposed). “C” is also true; all lesions are in a ings can be seen with a tricyclic overdose. In fact, the most
similar state of evolution. Finally, “D” is true and is the reason common presenting rhythm is a narrow-complex sinus tachy-
we do not currently immunize against smallpox—well, that and cardia. As toxicity progresses, you can see a prolonged PR
the fact we eradicated it in the wild (Way to go, humans!). interval, a widened QRS complex, and a prolonged QT inter-
val. A QRS >100 ms is predictive of seizures and QRS >160
ms is highly predictive of ventricular arrhythmia in patients
▶▶ CASE 1.2 with a tricyclic antidepressant overdose. Heart blocks (second-
and third-degree) herald a poor outcome and may be seen late
A 22-year-old female presents to the ED with an overdose. in the course. Asystole is not a primary rhythm in tricyclic
She has a history of depression, and there were empty bottles overdose and tends to reflect the end stage of another
found at her bedside. The bottles had contained clonazepam arrhythmia.
and nortriptyline. The patient is unconscious with dimin-
ished breathing and is unable to protect her airway.
YIKES!! The patient becomes unresponsive and you look at
the monitor. You obtain an ECG which shows the following
Question 1.2.1 The BEST next step is to:
(Fig. 1-1).
A) Intubate the patient
B) Begin gastric lavage and administer charcoal
Question 1.2.3 What is the patient’s rhythm?
C) Administer flumazenil, a benzodiazepine antagonist, to
A) Monomorphic ventricular tachycardia
awaken her and improve her respirations
B) Sinus tachycardia with a bundle branch block
D) Administer bicarbonate
C) Paroxysmal supraventricular tachycardia
E) Administer lipid emulsion
D) Torsades de pointes
E) Third degree heart block
Answer 1.2.1 The correct answer is “A.” This patient should
be intubated. Remember in any emergency situation that the Answer 1.2.3 The correct answer is “D.” This is torsades de
ABCs (airway, breathing, and circulation) are the priority. “B” is pointes which is a subtype of polymorphic ventricular tachycar-
incorrect because, as noted earlier, patients who undergo gastric dia. In French, it literally means “twisting of the points,” but in
lavage have a higher incidence of pulmonary aspiration—an even every language it means “bad news.” Torsades de pointes can be
greater concern in the obtunded patient. In fact, airway protection recognized by the varying amplitude of the complex in a some-
is MANDATORY before undertaking lavage. “C” is incorrect. what regular pattern. “A” is incorrect because the complexes are
Flumazenil will reverse the benzodiazepine. However, we know not monomorphic. “B” is incorrect for two reasons. First, there
from experience that seizures in patients who have had flumaze- are no P waves visible. Second, sinus tachycardia should not
nil are particularly difficult to control. This would be particularly have varied amplitude. “C” is incorrect because, again, there are
problematic in a patient with a mixed overdose, such as with a no P waves and the complexes are polymorphic. “E” is incorrect
tricyclic, where seizures are common. Thus, it is recommended because there are no P waves.
that flumazenil be used only as a reversal agent after procedural
sedation in patients who are not on chronic benzodiazepines. “E” Question 1.2.4 This patient needs treatment post haste.
is incorrect. Lipid emulsion refers to the liquid fatty acids given After taking care of the ABCs, what is the ONE BEST drug for
as part of total parenteral nutrition and theoretically can be used the treatment of this arrhythmia in a patient with a tricyclic
to bind fat-soluble drugs in the blood. Case series support con- overdose?
sideration of lipid emulsion for calcium channel blocker, beta- A) Esmolol
blocker, and tricyclic antidepressant overdoses, as well as other B) Lidocaine
fat-soluble drugs but only in cases of refractory cardiac arrest or C) Sodium bicarbonate
cardiovascular collapse—and certainly not before the airway has D) Procainamide
been secured. Keep reading for a discussion of answer “D.” E) Amiodarone

You notice that the patient begins to have an abnormal trac- Answer 1.2.4 The correct answer is “C.” The treatment of
ing on the cardiac monitor, so you order an ECG. choice for arrhythmias in patients with a tricyclic overdose is
sodium bicarbonate. Raising the pH and administering sodium
Question 1.2.2 Which of the following findings would you seem to “prime” the sodium channels in the heart, reversing
expect to find in a tricyclic overdose? the toxicity of the tricyclic. Procainamide (“D”) and quinidine
A) Normal QRS complex should not be used because they act in similar fashion to tricy-
B) Second- and third-degree heart block clics and may worsen the problem. Lidocaine (“B”) can be used
C) Widened QRS complex as can amiodarone (“E”), but they are not the best choices. Beta-
D) Sinus tachycardia blockers such as esmolol (“A”) can worsen hypotension and
E) Any of the above should be avoided.

Ch01.indd 3 29-11-2019 12:14:48


4 FAMILY MEDICINE EXAMINATION & BOARD REVIEW

FIGURE 1-1. ECG for patient in question 1.2.3.

phenytoin and fosphenytoin can cause hypotension—not what


This is not your patient’s lucky day. She begins to seize after you need in this unstable patient.
the administration of the bicarbonate.

Question 1.2.5 The treatment of choice for this seizing You correct the arrhythmia and stop the seizures, and she is
patient is: admitted to the intensive care unit.
A) Lorazepam (Ativan)
B) Repeat the bolus of sodium bicarbonate and start a bicar- HELPFUL TIP:
bonate drip A patient who is entirely asymptomatic 6 hours after a
C) Phenytoin (Dilantin) tricyclic overdose is unlikely to have any serious con-
D) Fosphenytoin (Cerebryx) sequences from the ingestion. They can be “medically
E) None of the above cleared” at that point for admission to a psychiatric
unit. Note that “symptomatic” may just be tachycardia
Answer 1.2.5 The correct answer is “A.” Benzodiazepines or mild confusion. We mean the entirely asymptomatic
are the treatments of choice in tricyclic-induced seizures. patient.
While most seizures are self-limited, it is important to control
seizures because the resultant acidosis can worsen tricyclic
toxicity (beyond the fact that prolonged seizures can cause ▶▶ Objectives: Did you learn to …
CNS injury). “B” is incorrect. This patient is already alkalinized, • Understand the importance of the ABCs in an unstable
and although sodium bicarbonate is the preferred therapy for patient?
tricyclic-induced cardiovascular toxicity, sodium bicarbonate • Describe the role of flumazenil in toxicologic emergencies?
is not particularly effective in tricyclic-induced seizures. “C,” • Manage a tricyclic overdose?
phenytoin, can be used, but benzodiazepines and phenobarbital • Recognize ECG findings in a tricyclic overdose?
should be administered first if possible. In addition to not work- • Recognize torsades de pointes and its treatment in the
ing well as an antiepileptic drug in tricyclic overdose, phenytoin context of a tricyclic overdose?
is also a class Ib antiarrhythmic, which may further prolong
the QRS and worsen the cardiac toxicity of the tricyclic. “D” is
incorrect for two reasons. First, since fosphenytoin is metabo- QUICK QUIZ: DESIGNER AND CLUB DRUGS
lized to phenytoin, the concern about efficacy applies. Second,
fosphenytoin is a prodrug and requires adequate circulation and An 18-year-old male presents after a party. He is having alter-
renal and hepatic function to be converted into active drug. If nating episodes of combative behavior interspersed with epi-
our patient becomes hypotensive with poor liver and renal per- sodes of coma. He becomes almost apneic during the episodes
fusion, adequate drug levels might not be achieved. Finally, both of coma. He has alternating bradycardia (while in coma) and

Ch01.indd 4 29-11-2019 12:14:48


CHAPTER 1 • Emergency Medicine 5

TABLE 1-1   TOXIDROMES


Drug Class Examples Signs and Symptoms
Anticholinergic Tricyclics, diphenhydramine, scopolamine, loco Tachycardia, flushing, dilated pupils, low-grade temperature,
weed (jimson weed), some mushrooms, etc. and confusion. Mnemonic: Dry as a bone, red as a beet, mad
as a hatter, blind as a bat

Opiates Morphine, heroin, codeine, oxycodone, etc. Pinpoint pupils, hypotension, hypopnea, coma, hypothermia

Cholinergic Organophosphate or carbamate pesticides, Lacrimation, salivation, muscle weakness, diarrhea, vomiting,
some mushrooms miosis. Mnemonic: SLUDGE BBB (salivation, lacrimation,
urination, diarrhea, GI upset, emesis … Bradycardia,
bronchorrhea, bronchospasm)

Sympathomimetic Cocaine, ecstasy, methamphetamine Tachycardia, hypertension, elevated temperature, dilated


pupils (mydriasis)

Gamma-hydroxybutyrate (GHB) GHB, liquid ecstasy, etc. Alternating coma with agitation, hypopnea while comatose,
bradycardia while comatose, and myoclonus

tachycardia when awake. The patient is also having myoclonic B) Bradycardia, pinpoint pupils, flushing, and decreased bowel
seizures. His serum alcohol level is zero, and his pupils are sounds
miotic. C) Tachycardia, dilated pupils, diaphoresis, and increased
bowel sounds
The most likely drug causing this is: D) Tachycardia, dilated pupils, flushing, and decreased bowel
A) Ecstasy (MDMA) sounds
B) GHB (gamma-hydroxybutyrate aka “liquid ecstasy”) E) Tachycardia, pinpoint pupils, flushing, and increased bowel
C) Methamphetamine sounds
D) LSD (lysergic acid diethylamine aka “acid”)
E) Oxycodone The correct answer is “D.” This patient has an anticholinergic
toxidrome. Toxidromes are symptom complexes associated with
The correct answer is “B.” The episodic coma and bradycar- a particular overdose that should be immediately recognized by
dia interspersed with episodes of extreme agitation are almost the clinician. Common toxidromes are listed in Table 1-1.
pathognomonic of GHB overdose. GHB intoxication also causes
pinpoint pupils. “A” is incorrect because MDMA causes an
amphetamine-like reaction with agitation, hypertension, hyper- ▶▶ CASE 1.3
thermia, tachycardia, etc. “C” is incorrect for the same reason. A patient presents to your office with neck pain after a motor
“D” is incorrect because LSD rarely (if ever) causes coma. “E” is vehicle accident. He was restrained and the airbag deployed.
incorrect because patients with opioid overdoses are generally He notes that he had some lateral neck pain at the scene. He
somnolent or comatose without interspersed episodes of agita- continues to have lateral neck pain.
tion, although opioids may also cause miosis (be aware that not
all narcotic overdoses are associated with pinpoint pupils). GHB Question 1.3.1 Which of the following IS NOT a criterion for
is odorless and has slight salty taste. Besides being a street drug, clearing the cervical spine clinically?
GHB is available by prescription as “sodium oxybate” for nar- A) Absence of all neck pain
colepsy (Xyrem). It has become a drug of choice for “date rape” B) Normal mental status including no drugs or alcohol
since it cannot be detected in the urine. The toxicity tends to be C) Absence of a distracting injury (such as an ankle fracture)
self-limited and can be treated with intubation if needed along D) Absence of paralysis or another “hard” sign that could be
with tincture of time. The half-life is only 27 minutes. caused by a neck injury
E) Absence of retrograde amnesia

QUICK QUIZ: TOXIDROMES Answer 1.3.1 The correct answer is “A.” Patients can have lat-
eral neck pain and still have their cervical spines cleared clini-
A patient presents to the hospital with a diphenhydramine cally. However, no one will fault you for obtaining radiographs
overdose. in patients with lateral muscular (e.g., trapezius) neck pain.
Patients with central neck pain (e.g., over the spinous processes)
Which of the following signs and symptoms are you likely DO need imaging (radiographs ± CT) to clear their cervical
to find in this patient? spine. All of the other criteria are required in order to clinically
A) Bradycardia, dilated pupils, flushing, and increased bowel clear the cervical spine (Table 1-2). These criteria have been
sounds validated in both adult and adolescent patients.

Ch01.indd 5 29-11-2019 12:14:49


6 FAMILY MEDICINE EXAMINATION & BOARD REVIEW

TABLE 1-2   CLEARING THE CERVICAL SPINE CLINICALLY


No central neck pain on questioning or palpation

No distracting, painful injury (e.g., bone fracture)

No symptoms or signs referable to the neck (paralysis, stinger-type injury, etc.)

Normal mental status including no drugs or alcohol. This includes any retrograde amnesia, etc.

syndrome (progressive numbness and weakness from autoim-


HELPFUL TIP:
mune myelitis).
CT has pretty much replaced plain x-rays in the evalu-
ation of the bones of the neck. If you are doing plain
Question 1.3.3 The next step in the management of this
films, the most common cause of missed fractures is an
patient is:
inadequate series of radiographs. An adequate series of
A) Avoid hypotension and hypoxia to prevent secondary insult
radiographs for the cervical spine includes an AP film,
to the cord
a lateral film including the top of T-1, and an odontoid
B) Fluid restriction and diuretics to reduce cord edema
film. Flexion–extension views add little and should be
C) Mannitol to reduce cord edema
avoided.
D) Neurosurgical intervention to decompress the cord
E) Lollipop and a gift card for “service recovery”

The patient’s daughter, aged 4 years, was in the same motor Answer 1.3.3 The correct answer is “A.” Patients with a cord
vehicle accident and also had her cervical spine cleared by injury should be monitored closely to avoid hypotension and
radiograph. However, you get a call from the ED 48 hours hypoxia, both of which will further damage the already com-
after the initial accident that she is paralyzed from just above promised spinal cord. Neither diuretics (“B”) nor mannitol
the nipple line down (never a good thing—you quickly make (“C”) will be useful in this situation. “D” is incorrect because
a mental note to make sure your malpractice insurance pre- the process of SCIWORA involves stretching of the cord (and
miums are paid up). You review the initial radiographs with subsequent dysfunction) rather than cord compression such as
the radiologist, which are negative as is a CT of the cervical would be seen with a bony injury. “E” might be the right choice
spine bones done after the onset of the paralysis. if you are taking this test as a “patient experience expert” instead
of a doctor; but doctors should choose “A.”
Question 1.3.2 The most likely cause of this patient’s
paralysis is:
A) Missed transection of the thoracic cord HELPFUL TIP:
B) Conversion reaction from the psychological trauma of the Don’t use steroids for spinal cord injuries. It doesn’t
accident work. There are also secondary complications from
C) Subarachnoid hemorrhage the steroids, including hyperglycemia, myopathy, and
D) SCIWORA syndrome infections (e.g., pneumonia).
E) Guillain–Barre syndrome

Answer 1.3.2 The correct answer is “D.” This likely represents Question 1.3.4 The father is, understandably, irate that his
SCIWORA syndrome (spinal cord injury without radiologic child is now paralyzed. You can tell him that the natural
abnormality). SCIWORA has become a bit of a misnomer history of SCIWORA syndrome in THIS CHILD is likely to be
in the age of MRI, as up to two-thirds of children with this the following:
diagnosis will have abnormal MRI findings. This entity occurs A) Continued paralysis with the necessity of long-term, perma-
from stretching of the cord secondary to flexion/extension- nent adaptation to the injury
type movement in an accident. Patients with SCIWORA syn- B) Progression of the injury over the next week to include fur-
drome may be paralyzed at the time of initial presentation (in ther paralysis in an ascending fashion
the event of cord transection) or may have a delayed presenta- C) Resolution of paralysis and sensory symptoms over the next
tion up to 72 hours after the injury. “A” is incorrect because a several months
cord transection would present with paralysis immediately at D) Resolution of all symptoms except sensory symptoms over
the time of injury. “B” is incorrect because this child is 4 years the next several months
old, and conversion reaction is unlikely in children. In addi- E) Large lawsuit payout on the way. Do not pass go; do not col-
tion, conversion reaction is always a diagnosis of exclusion. lect $200; go directly to a malpractice attorney
“C” and “E” are incorrect because this is neither the presen-
tation of a subarachnoid hemorrhage (headache, stiff neck, Answer 1.3.4 The correct answer is “C.” Generally, patients
perhaps focal neurologic symptoms) nor of Guillain–Barre with SCIWORA syndrome regain their strength and sensory

Ch01.indd 6 29-11-2019 12:14:49


CHAPTER 1 • Emergency Medicine 7

abilities over time. However, this depends on when they pres- Question 1.4.2 This patient’s anion gap is:
ent with symptoms! Patients who present with paralysis right A) 13
after the accident may have complete cord transection and B) 15
thus will not regain function. For this reason, it is important to C) 23
obtain an MRI on all patients with SCIWORA syndrome (and D) Unable to calculate the anion gap with the information
any trauma-induced paralysis for that matter). Patients with provided
significant spinal cord findings on MRI are more likely to have
persistent deficits. Answer 1.4.2 The correct answer is “B.” By convention, the
anion gap is calculated without using a major cation, potassium.
▶▶ Objectives: Did you learn to …

Thus, the anion gap is calculated as follows:
Clinically “clear” the cervical spine and decide when to order
cervical spine radiographs?
• Understand the physiology, natural history, and sodium − (chloride + bicarbonate)
management of SCIWORA syndrome?
In this patient, the anion gap = 135 - (108 + 12) = 15.
The normal anion gap is typically considered to be 12 or
▶▶ CASE 1.4 less. However, since albumin is the major unmeasured anion
A patient with an extensive history of alcohol use presents to in the serum, the anion gap should be adjusted for hypoal-
the ED after drinking a bottle of automobile winter gas treat- buminemia. Every 1 g decrease in albumin will decrease the
ment (Rothschild Vintage, 1954). He is intoxicated, has a anion gap by about 3. Therefore, you should subtract (3 ×
headache, and describes a “misty” vision, “like a snowstorm” [normal albumin − actual albumin]) to get the “real” anion
(if you live in southern Florida or Hawaii, call one of us in gap. The normal albumin is considered to be 4. So, let us say
Iowa for a description). He is tachycardic and tachypneic. we calculate an anion gap of 16 but the albumin is 2. In this
You start an IV and administer saline. You obtain a blood case, the corrected anion gap will be (16 − [3 × (4 − 2)] =
gas, which shows a mild metabolic acidosis. 16 − 6), or 10.

Question 1.4.1 A metabolic acidosis is consistent with all


of the following ingestions EXCEPT: HELPFUL TIP:
A) Ethylene glycol In methanol ingestions, the severity of acid–base distur-
B) Methanol bance is generally a better predictor of outcome than
C) Ethanol (e.g., vodka, gin) serum methanol levels.
D) Petroleum distillates (e.g., non-alcohol-containing gasoline
products)
Question 1.4.3 All of the following are causes of an anion
Answer 1.4.1 The correct answer is “D.” Ethylene glycol, gap acidosis EXCEPT:
methanol, and ethanol can all cause a metabolic acidosis. A) Lactic acidosis
Hydrocarbons (e.g., gasoline products) do not cause a meta- B) Diabetic ketoacidosis
bolic acidosis. The main manifestation of hydrocarbon toxicity C) Renal tubular acidosis
is secondary to the inhalation and aspiration of the hydrocar- D) Uremia
bon and the resulting pneumonitis. E) Ingestions such as methanol

This patient’s electrolytes are as follows: sodium 135 mEq/L, Answer 1.4.3 The correct answer is “C.” See Table 1-3 for more
bicarbonate 12 mEq/L, chloride 108 mEq/L, BUN 12 mg/dL, on causes of anion gap acidosis.
Cr. 1 mg/dL.

TABLE 1-3   CAUSES OF ACIDOSIS


Causes of an elevated anion gap acidosis Lactic acidosis
Diabetic ketoacidosis
Ingestions such as ethanol, methanol, etc.
Uremia
Alcoholic ketoacidosis

Causes of a normal anion gap acidosis GI bicarbonate loss (e.g., chronic diarrhea)
Renal tubular acidosis (types I, II, and IV)
Interstitial renal disease
Ureterosigmoid loop
Acetazolamide and other ingestions
Small bowel drainage

Ch01.indd 7 29-11-2019 12:14:50


8 FAMILY MEDICINE EXAMINATION & BOARD REVIEW

Question 1.4.4 Which of the following findings IS NOT know that the patient did not simply overindulge on ethanol
frequently seen in patients with methanol ingestion? (methanol, such as “gas dry,” will make one intensely drunk …
A) Hypopnea but has obvious downsides).
B) Optic disk abnormalities
C) Abdominal pain and vomiting You decide that there is sufficient evidence that this patient
D) Basal ganglia hemorrhage has ingested methanol to institute treatment.
E) Meningeal signs, such as nuchal rigidity
Question 1.4.6 Appropriate treatment(s) for this patient
Answer 1.4.4 The correct answer is “A.” Hypopnea is not include:
commonly seen in methanol poisoning until the patient is close A) Fomepizole (4-MP)
to death. In fact, the reverse is true. Tachypnea is a frequent B) Acetylsalicylic acid
finding in methanol overdose. This makes sense. The patient C) Ethanol
is trying to compensate for a metabolic acidosis by blowing off D) A and C
CO2. Optic disk abnormalities, abdominal pain and vomiting, E) All of the above
basal ganglia hemorrhage, and meningeal signs are all seen as
part of methanol toxicity. It is thought that many of these signs
Answer 1.4.6 The correct answer is “D.” Both fomepizole
and symptoms are secondary to central nervous system (CNS)
(4-MP) and ethanol are used for methanol ingestion. The idea
hemorrhage.
is to slow down the metabolism of the methanol. The toxicity
of methanol is caused by formic acid, which is a by-product
You can test for ethanol at your hospital but do not have a of methanol metabolism. Ethanol is metabolized by alcohol
test for methanol on a stat basis and want to be sure that this dehydrogenase, the same enzyme that breaks down methanol.
patient is not just saying he has a methanol ingestion in order Thus, methanol metabolism is competitively inhibited by etha-
to obtain alcohol (a treatment for methanol ingestion—break nol. The same holds true for fomepizole, which is a competitive
out the single malt scotch!). inhibitor of alcohol dehydrogenase. Fomepizole and ethanol
can both be used for ethylene glycol ingestion as well. “B” is
Question 1.4.5 What test is most likely to help you incorrect. Acetylsalicylic acid (ASA), or aspirin, has no role in
determine if the patient has methanol ingestion? methanol ingestion, and would likely worsen any gastritis or
A) Complete blood cell count (CBC) hemorrhaging.
B) BUN/creatinine
C) Liver enzymes
D) Measured serum osmolality HELPFUL TIP:
E) Amylase and lipase Hemodialysis should be available for any patient who
has ingested methanol. Indications for hemodialysis
Answer 1.4.5 The correct answer is “D.” With a mea- include methanol level >50 mg/dL, severe and resistant
sured serum osmolality, you can calculate the osmolar gap. acidosis, and renal failure.
To do so, subtract the total measured serum osmoles from
the osmoles known to be due to ethanol (each 100 mg/dL
of ethanol accounts for approximately 22 osmoles). If there ▶▶ Objectives: Did you learn to …
• Recognize manifestations of alcohol ingestion?
is an elevated osmolar gap, it is evidence of a circulating,
unmeasured osmole. A normal osmolar gap is usually about • Identify causes of metabolic acidosis with elevated and
6 mOsm/L or less. Any osmolar gap over 10 mOsm/L sug- normal anion gaps?
gests a concurrent ingestion with the alcohol (such as in our • Use the osmolar gap to narrow down the differential
case, methanol). diagnosis of metabolic acidosis?
In this case, for example:
Measuredserumosmolality = 368
QUICK QUIZ: BETA-BLOCKER OVERDOSE
Bloodalcohol = 200 mg/dLor about 44 osmoles
Which of the following has been shown to be useful in
Calculated osmolality = 2(Na) + BUN/2.8 + glucose/18 beta-blocker overdose when conventional, adrenergic
vasopressors are ineffective?
= 280 + 6 + 8 = 294
A) Calcium chloride
So,osmolar gap = 368 − (294 + 44) = 30 B) Glucagon
C) Milrinone
This means that there are 30 unmeasured osmoles that could, D) High-dose insulin
in the clinical context of the case, represent methanol. Thus, we E) All of the above

Ch01.indd 8 29-11-2019 12:14:53


CHAPTER 1 • Emergency Medicine 9

The correct answer is “E.” In beta-blocker overdoses, the


following findings may be observed: bradycardia, AV block, QUICK QUIZ: TOXICOLOGY 2
hypotension, bronchospasm, nausea, emesis, and hypoglyce-
mia. This is very similar to the presentation of a calcium chan- The best therapy for seizures secondary to an isoniazid
nel blocker overdose, but calcium channel blocker overdose overdose is:
often lacks bronchospasm and patients are hyperglycemic as A) Lorazepam
insulin release from the islet cells is calcium dependent. When B) Phenytoin
a beta-blocker overdose has been identified, the usual treat- C) Pyridoxine
ments are employed (e.g., IV fluids, vasopressors, airway pro- D) Thiamine
tection). If conventional vasopressors have failed, glucagon E) Phenobarbital
in a dose of 3 to 5 mg IV bolus and a drip at 1 to 5 mg/hr
may be effective in treating beta-blocker overdose. It is gener- The correct answer is “C.” Isoniazid is a vitamin B6 antagonist.
ally preferred over atropine in this situation. Milrinone and Thus, pyridoxine (in massive doses!) is the drug of choice in
other phosphodiesterase inhibitors may also be used but are isoniazid-induced seizures. These seizures are often resistant to
considered third-line agent. Likewise, calcium is considered conventional therapy. Look for this type of overdose in patients
a third-line agent in beta-blocker overdose. Calcium chloride who are being treated for active or latent tuberculosis; often
may potentiate the action of glucagon. There is also a grow- these patients are immigrants and were exposed to TB in their
ing body of literature supporting use of high-dose insulin country of origin.
(while maintaining euglycemia) for beta-blocker, calcium
channel blocker, or combination overdoses. The insulin is
started at 0.5 U/kg/hr and titrated to as high as 10 U/kg/hr, ▶▶ CASE 1.5
titrated by hemodynamic improvement (Clin Toxicol (Phila).
2011;49(4):277–283). Dextrose infusion may be required to A family of four comes into your ED after being exposed to car-
maintain euglycemia. Does it seem counterintuitive to give bon monoxide (CO). They were in an idling car in the garage
glucagon and insulin? Both appear to mitigate the deleteri- and were running the engine and heater to stay warm. You
ous effects of beta-blockers on cardiac myocyte metabolism, want to get a carboxyhemoglobin level on the whole family,
although the mechanism is not completely understood. Both but cannot get an arterial blood gas from the youngest child.
can be given (usually in sequence with the glucagon first) to
the same patient for beta-blocker overdose. Remember to Question 1.5.1 What is your response?
maintain euglycemia if using insulin for beta-blocker or A) Check pulse oximetry, and if the oxygen saturation is nor-
calcium channel blocker overdose. mal, be reassured
B) Check end-tidal carbon dioxide
C) Check a venous carboxyhemoglobin level
D) Check a venous carboxyhemoglobin and correct for the dif-
QUICK QUIZ: TOXICOLOGY 1 ference between venous and arterial samples

Which of the following can be used to increase the Answer 1.5.1 The correct answer is “C.” A venous carboxyhe-
metabolism of alcohol in an intoxicated patient? moglobin is just as accurate as an arterial c­ arboxyhemoglobin—
A) IV fluids in fact, no correction is needed, which is why “D” is wrong—and,
B) Charcoal a venous gas much less painful to draw. “A” is incorrect because
C) Forced diuresis the pulse oximeter does not reflect hypoxia in carbon monoxide
D) GABA antagonists such as flumazenil poisoning. Thus, standard pulse oximetry is useless in deter-
E) None of the above mining the carboxyhemoglobin level. “B” is incorrect because
end-tidal carbon dioxide is measuring CO2 and not CO.

The correct answer is “E.” Drunk patients, no matter how Question 1.5.2 When determining which patients need
much they annoy you, will just have to sleep it off. The rate of hyperbaric oxygen on the basis of a carboxyhemoglobin
alcohol metabolism is fixed with zero-order kinetics at lower level, the level to rely upon is:
doses (fixed metabolic rate) and first-order kinetics at higher A) The carboxyhemoglobin level on arrival to the ED
doses (rate proportional to levels). In general, this rate is in the B) The carboxyhemoglobin level at 4 hours after exposure
range of 9 to 36 mg/dL/hr, with 20 mg/dL/hr being the accepted C) The carboxyhemoglobin level projected to “time zero” (e.g.,
norm. At this point, there are no available agents to increase the at the time of exposure)
metabolism of ethanol. “B” is incorrect because ethanol is too D) None of the above
rapidly absorbed for charcoal to be of any benefit. “C,” forced
diuresis, will just result in an incontinent, sleeping patient and Answer 1.5.2 The correct answer is “C.” A major consider-
does not hasten metabolism. ation regarding the initiation of hyperbaric oxygen therapy is

Ch01.indd 9 29-11-2019 12:14:53


10 FAMILY MEDICINE EXAMINATION & BOARD REVIEW

the patient’s clinical situation. More severely ill patients with oxygen in the appropriate patient can reduce long-term neuro-
CO poisoning (e.g., severe acidosis, unconscious, unresponsive) logic sequelae.
should be considered candidates for hyperbaric oxygen, and
some hyperbaric oxygen centers will treat regardless of mea- Your closest diving chamber is about 90 minutes away and
sured carboxyhemoglobin level in these patients. If the treat- will hold only one patient at a time. You need to make a deci-
ment decision is made based on the carboxyhemoglobin level, sion about who to send for hyperbaric oxygen.
the level projected to time zero gives the most accurate informa-
tion about the degree of exposure. The rest of the answers are Question 1.5.5 Which patient will benefit most from
incorrect. hyperbaric oxygen therapy?
A) Asymptomatic pregnant mother, time zero carboxyhemo-
The father has a headache and a time zero carboxyhemoglo- globin of 18%
bin level of 12%. The mother, who is pregnant, is asymptom- B) Asymptomatic 6-year-old, time zero carboxyhemoglobin of
atic and has a time zero carboxyhemoglobin level of 18%. 18%
Both of the children are asymptomatic. The 6-year-old has a C) Asymptomatic 8-year-old, time zero carboxyhemoglobin of
time zero carboxyhemoglobin level of 18% while the 8-year- 23%
old has a level of 23%. D) Adult male with mild headache only, time zero carboxyhe-
moglobin level of 12%
Question 1.5.3 The first step in the treatment of these
patients is: Answer 1.5.5 The correct answer is “A.” Generally accepted crite-
A) Start an IV and administer saline ria for hyperbaric oxygen include: mental status changes, asymp-
B) Start N-acetylcysteine, which is a free radical scavenger tomatic carboxyhemoglobin levels >25%, acidosis, cardiovascular
C) Start continuous positive airway pressure (CPAP) to maxi- disease, and age >60. Obviously, these are relative criteria. An
mize airflow by keeping the airways from collapsing otherwise normal 61-year-old with a mild exposure need not have
D) Administer 100% oxygen hyperbaric oxygen. Pregnancy is an indication for hyperbaric oxy-
E) Intubate the most severe patient, 100% oxygen for the others gen therapy because fetal hemoglobin has a high affinity for carbon
monoxide, with the fetus acting as a “sink” for CO. The high aerobic
Answer 1.5.3 The correct answer is “D.” Because CO competi- metabolic activity of fetal development is impacted greatly by expo-
tively binds to hemoglobin in place of oxygen and in fact has sure to the anaerobic environment created by carbon monoxide.
greater affinity for hemoglobin than oxygen, high-flow 100%
oxygen is the cornerstone of treating CO poisoning. The half- Question 1.5.6 All of the following are well-established
life of CO in a patient breathing room air is approximately 300 consequences of hyperbaric oxygen EXCEPT:
minutes; this is reduced to 90 minutes when breathing high A) Seizures
flow oxygen and reduced to 30 minutes when breathing 100% B) Psychosis
hyperbaric oxygen. Thus, the first step in CO poisoning is to C) Myopia
administer 100% oxygen. The rest of the answers are incorrect. D) Ear and pulmonary barotraumas
If the patient is not ventilating well and requires intubation, E) Direct pulmonary oxygen toxicity
this would be appropriate, and the FiO2 should be set to 100%,
regardless of the patient’s pulse oximetry or arterial oxygen Answer 1.5.6 The correct answer is “B.” All of the rest are
readings. However, in our patients who are breathing without found as a result of hyperbaric oxygen. “C,” myopia, is found in
difficulty, there will be no advantage (and much higher risk) to up to 20% of patients being treated with hyperbaric oxygen. It is
intubation. due to direct toxicity of oxygen on the lens and usually recovers
within weeks to months.
Question 1.5.4 Which of the following can be seen with
carbon monoxide poisoning? ▶▶ Objectives: Did you learn to …
• Diagnose and manage patients with carbon monoxide
A) Rhabdomyolysis
poisoning?
B) Cardiac ischemia
• Describe complications of carbon monoxide poisoning?
C) Long-term neurologic sequelae, including dementia
• Identify patients who may benefit from hyperbaric oxygen
D) Pulmonary edema
E) All of the above therapy?
• Describe the complications of hyperbaric oxygen therapy?
Answer 1.5.4 The correct answer is “E.” All of the above can
be seen with carbon monoxide poisoning. Additional findings
include lactic acidosis, seizures, syncope, and headache. “C” ▶▶ CASE 1.6
deserves a bit more discussion. Long-term neurologic sequelae
can develop from days to months after the exposure and include A 50-year-old man comes to your ED after being bitten by a
cognitive deficits, focal neurologic deficits, movement disor- stray dog outside your hospital. Apparently, there is a prob-
ders, and personality changes. It appears that using hyperbaric lem with roving packs of feral dogs in your part of town. The

Ch01.indd 10 29-11-2019 12:14:53


CHAPTER 1 • Emergency Medicine 11

bite was unprovoked and is on the abdomen. The patient has be isolated for 10 days, not 3. “B” is incorrect. If captured, the
no other health history of note and has not taken antibiotics animal can be sacrificed, but the brain should be examined—
for over a year. There is a 3-cm laceration on the abdomen. not the liver. “C” and “D” are both incorrect methods of admin-
istering the vaccine and immune globulin. Note that the average
Question 1.6.1 All of the following are true about dog bites incubation period of rabies is 85 days. So, immunizing up to
EXCEPT: 3 months (90 days) after the bite-event is indicated.
A) They tend to be primarily crush-type injuries
B) In general, the infection rate is similar to a laceration from Question 1.6.3 Which of the following requires rabies
any other mechanism (e.g., knife cut), except on the hands prophylaxis in all cases?
and feet A) Stray rabbit bites
C) A common organism in infected dog bites is Staphylococcus B) Stray rat bites
aureus C) Stray bat bites
D) Primary closure of dog bite wounds is an acceptable option D) Stray squirrel bites
(except perhaps on the hands and feet) E) Stray snake bites
E) They always require antibiotics
Answer 1.6.3 The correct answer is “C.” All bats should be con-
Answer 1.6.1 The correct answer is “E.” All of the rest are true sidered rabid unless available for observation and testing. See
statements. Dog bites (except, perhaps, for those from teacup Table 1-4 for detailed recommendations. Also, see the CDC or
poodles named Fifi) tend to be crush injuries (as contrasted your state public health website for information about rates of
with cat bites and Fifi, which are primarily puncture wounds). infection in wild animals in your area. Of note, there have been
The infection rate is about the same as other lacerations. Bites rabid squirrels in Iowa City, our illustrious home: Google it.
on the hands and feet tend to have a higher rate of infection.
Most dog bite infections are polymicrobial with mixed aerobic HELPFUL TIP:
and anaerobic bacteria. S. aureus is often present, along with Patients should receive a tetanus booster every 10
other organisms including Pasteurella and Capnocytophaga (say years. For a contaminated wound, the tetanus booster
that one ten times fast). Other organisms include Streptococcal should be within the last 5 years. Patients should
species and Gram-negative species. Dog bites do not generally receive at least one dose of Tdap (tetanus, diphtheria,
require antibiotic prophylaxis, except under certain circum- and acellular pertussis) between ages 11 and 18 and a
stances (e.g., presentation >9 hours after bite, immunocompro- single dose between ages 18 and 64. In addition, health-
mised, large, or complicated wound, perhaps hands and feet). care workers and those > 65 years of age who will be
around infants should receive a single dose of Tdap (for
Question 1.6.2 You are concerned about rabies prophylaxis. pertussis prophylaxis). Pregnant women should have a
Which of the following is the best next step? Tdap with every pregnancy to protect the infant (week
A) Isolate the suspect animal for 3 days 27–30 seems optimal; Healy CM et al. JAMA 2018 Oct 9).
B) Sacrifice the suspect animal and examine the liver
C) Administer rabies immune globulin IM
D) Administer rabies immune globulin IV followed by rabies HELPFUL TIP:
vaccination series If a patient at risk for tetanus has not had a primary
E) Administer rabies immune globulin by infiltrating it around series of tetanus immunizations, administer tetanus
the wound followed by rabies vaccination series immune globulin, and start the primary tetanus series.
Children who have had at least 3 doses of their primary
Answer 1.6.2 The correct answer is “E.” You should infiltrate series (routinely given at age 2, 4, and 6 months) are
rabies immune globulin around the wound and then begin the considered “immune” and can be given a booster as
rabies vaccination series. Infiltrate as much of the immune glob- needed as listed above. DTaP is appropriate for children
ulin as possible around the wound and administer the remainder aged 6 weeks to 7 years.
IM at a different site. Do not give more than the recommended
dose of immunoglobulin. This can reduce the immunogenic-
ity of the vaccine. “A” is incorrect because the animal needs to You decide to irrigate this patient’s wound.

TABLE 1-4   GUIDELINES FOR RABIES PROPHYLAXIS


General Rule Animals
Always assume rabid unless available for testing Foxes, bats, raccoons, skunks, dogs, cats, ferrets, other carnivores

Judge on an individual basis Rodents (rats, mice, etc.), lagomorphs (rabbits, etc.), squirrels

Never require rabies prophylaxis Non-mammals (snakes, lizards, etc.)

Ch01.indd 11 29-11-2019 12:14:53


12 FAMILY MEDICINE EXAMINATION & BOARD REVIEW

Question 1.6.4 Which of the following statements is true • Use various wound irrigation solutions for cleansing
about irrigating a wound and subsequent risk of wound wounds?
infections? • Decide upon the time frame for wound closure?
A) Povidone-iodine as a 50% irrigation solution (e.g., Betadine)
in the wound will decrease the infection rate
B) Irrigation with normal saline is the only recommended ▶▶ CASE 1.7
method of cleaning a wound
C) Irrigation with normal saline and irrigation with tap water A 52-year-old male presents to your ED via ambulance
are equally effective in reducing wound infection rates complaining of a headache after a fall. He was working
D) Use of lidocaine with epinephrine in a wound increases the and fell approximately 10 ft. He notes no injury except for
rate of infection head and neck pain. A quick survey reveals that he has a
E) Irrigation of a wound with either alcohol or hydrogen per- BP of 128/86 mm Hg, pulse 100 bpm, and respirations of
oxide will reduce the rate of wound infection 12. There was no loss of consciousness at the scene. He “saw
stars” and was clumsy, dazed, and cognitively slowed at the
Answer 1.6.4 The correct answer is “C.” Infection rates (in the scene without any focal neurologic deficit. He is now back
United States) are the same whether the wound is irrigated with to his baseline.
normal saline or tap water. “A” is incorrect. Povidone-iodine is
toxic to tissue and polymorphonuclear leukocytes and actually Question 1.7.1 A concussion is defined as:
may increase infection rates unless a solution of 1% or less is A) Any neurologic symptoms (e.g., clumsy, dazed, or slow, nau-
used. Full strength povidone-iodine can be used on intact skin sea, dizziness) after head injury
as a cleanser but should not be used in a wound. “B” is incor- B) Loss of consciousness followed by return to baseline
rect because other solutions (poloxamer 188, balanced salt solu- C) Loss of consciousness with continued neurologic
tions, etc.) can be used but are more expensive and do not offer symptoms
any benefit in reduction in infection rates. “D” and “E” are both D) Confusion after head trauma regardless of whether the
incorrect. Use of lidocaine with epinephrine (“D”) may be war- patient lost consciousness or not
ranted for local anesthesia for further wound exploration and/or E) Any traumatic injury to the head
closure. It has not shown to increase rates of infection. As with
povidone-iodine, alcohol (“E”) may be used for cleaning skin, Answer 1.7.1 The correct answer is “A.” A concussion is
but should be kept out of the wound. It is toxic to tissue and acts defined as any neurologic symptom after head trauma. Note
as a fixative. Hydrogen peroxide (“E”) is also toxic to tissue and that a concussion does not require a loss of consciousness. For
should not be used in open wounds—no matter what grandma this reason, “B” and “C” are incorrect. “D” is incorrect because
says! Chlorhexidine can also be used on intact skin (not in the manifestations of concussion are not limited to confusion, but
wound) and is more bactericidal than Povidone-Iodine. also include protracted vomiting, transient amnesia, slowed
mentation, “dizziness,” and other neurologic symptoms. “E” is
Question 1.6.5 How long after a laceration occurs can the incorrect because by definition, a concussion requires neuro-
wound be closed primarily? logic symptoms.
A) 6 hours
B) 12 hours Your patient opens his eyes spontaneously, follows com-
C) 18 hours mands, answers all orientation questions correctly, but
D) 24 hours appears unsteady when ambulating.
E) Any of the above can be correct depending on the wound
Question 1.7.2 His Glasgow Coma Scale (GCS) is:
Answer 1.6.5 The correct answer is “E.” There is no arbitrary A) 5
time limit to when a wound can be closed. Facial wounds may B) 10
be closed up to 24 hours after injury for cosmetic reasons, while C) 14
you may not want to close other, contaminated wounds more D) 15
than 12 hours after injury. Some wounds you may not want E) 20
to close at all (e.g., bites to the hand, wounds contaminated
with grease, wounds contaminated with manure, human bite
Answer 1.7.2 The correct answer is “D.” The Glasgow Coma
wounds), rather allowing them to close by secondary intention.
Scale (GCS) is a scale used to indicate the severity of neuro-
▶▶ Objectives: Did you learn to … logic dysfunction and is often applied to victims of head trauma.
• Describe the indications for rabies prophylaxis? Remember, however, that it does not predict mortality or mor-
• Recognize the issues that arise with animal bites and bidity, but is only used as a descriptive scale of the patient’s
indications for closure and/or prophylactic antibiotics? current state. Only the maximum score of 15 is considered a
• List recommendations for tetanus prophylaxis and normal GCS. There are three components to the GCS, listed in
boostering? Table 1-5.

Ch01.indd 12 29-11-2019 12:14:53


CHAPTER 1 • Emergency Medicine 13

TABLE 1-5   GLASGOW COMA SCALE


Eye opening Spontaneous = 4
Mnemonic: “4 eyes” To speech = 3
To pain = 2
No response = 1

Verbal response Alert and oriented = 5


Mnemonic: “Jackson 5” Disoriented conversation = 4
Nonsensical speech = 3
Moaning = 2
No response = 1

Motor response Follows commands = 6


Mnemonic: “Six Cylinders” Localizes pain = 5
Withdraws from pain = 4
Decorticate flexion = 3
Decerebrate extension = 2
No response = 1

Answer 1.7.4 The correct answer is “D.” Older patients are at


HELPFUL TIP:
greater risk of developing serious intracranial injuries, and the
Your chair and refrigerator each have a GCS of 3.
age of 60 is usually considered an independent indication for a
Remember that nothing can have a GCS less than 3.
head CT with significant injury. While there is no “upper limit
of normal” for vomiting after head trauma, the best data avail-
Question 1.7.3 In patients with head injury, and able suggest that any vomiting after head trauma in an adult
independent of other factors, a GCS score of 15 indicates indicates the need for a head CT. The currently recommended
that: criteria for a CT of the head in various age groups are listed in
A) The patient does not require a head CT scan Table 1-6.
B) There is essentially no possibility that this patient has an
intracranial injury requiring surgical intervention You obtain the head CT and find a subdural hematoma. You
C) There is little or no possibility that this patient has any focal arrange to transfer this patient for neurosurgical interven-
intracranial bleed tion in order to drain the subdural hematoma. It is about a
D) There is up to a 4% chance this patient will need neurosurgi- 4-hour drive by ambulance to the nearest facility that has a
cal intervention neurosurgeon.
E) None of the above
Question 1.7.5 Which of the following is indicated as
Answer 1.7.3 The correct answer is “D.” In appropriately prophylaxis against increased intracranial pressure in this
selected patients (e.g., those with a significant mechanism of patient?
injury), about 18% with a GCS of 15 will have some intracranial A) Hyperventilation after intubation
lesion, and up to 4% will eventually require neurosurgical inter- B) IV mannitol
vention. These are generally patients who have a depressed skull C) Trendelenburg position
fracture but a normal GCS. “A” is incorrect since a normal GCS D) IV dexamethasone
in and of itself does not allow one to forgo head CT in patients E) None of the above
with a significant mechanism of injury. “B” and “C” are also
incorrect for the reasons noted earlier. Remember that the GCS Answer 1.7.5 The correct answer is “E.” None of the above is
is not linear; a GCS of 14 is bad. Patients with a GCS of 14 must indicated as prophylaxis for increased intracranial pressure. “A”
have a CT scan—unless another factor in the clinical decision- is incorrect for two reasons. First, this patient does not need to
making dictates otherwise (e.g., the finding is preexisting from be intubated. Second, routine hyperventilation as prophylaxis
dementia or otherwise). for increased intracranial pressure is of no benefit. This has been
well studied. What happens is that hyperventilation does cause
Question 1.7.4 In an adult patient with a significant head vasoconstriction reducing intracranial blood flow and therefore
injury, which of the following is NOT an indication for a intracranial pressure. However, hyperventilation also causes
head CT scan? ischemia around the area of the injury secondary to the vaso-
A) Intoxication with drugs or alcohol constriction and may worsen outcomes. “B” is incorrect because
B) Persistent vomiting prophylactic mannitol, like prophylactic hyperventilation, con-
C) Amnesia or memory deficit fers no benefit. “C” is incorrect. Trendelenburg positioning, or
D) Age greater than 40 elevating the legs above the heart, would result in increased
E) Seizure intracranial pressure. There is very limited data to support or

Ch01.indd 13 29-11-2019 12:14:53


14 FAMILY MEDICINE EXAMINATION & BOARD REVIEW

TABLE 1-6   INDICATIONS FOR HEAD CT BY AGE


Indications for Head CT after Trauma with a Significant
Patient Age Mechanism
Adult Intoxication
Age >60
Any memory deficit
Vomiting (number of times undefined)
Seizure
Headache

PECARN Rules for Pediatric Head Trauma and Need for CT Scan (The Lancet. 2009;374(9696):1160–1170)

CT needed if:

Children >2 year GCS <15


Signs of a basilar skull fracture
Agitation, somnolence, slow responses, or perseveration
Vomiting
Loss of consciousness
Severe headache
Severe mechanism (Fall >5 ft, MVC with ejection, roll over, fatality)
Bike/pedestrian vs. car without helmet
High impact object
If there are changes during observation (MS change, worsening
headache, new or persistent vomiting), consider CT based on clinical
judgment

Children <2 years GCS <15


Palpable skull fracture
Agitation, somnolence, slow responses, perseveration
Scalp hematoma (except for frontal)
LOC >5 seconds
Severe mechanism (fall >3 ft, MVC with ejection, roll over, fatality)
Bike/pedestrian vs. car without helmet
High impact object
If there are changes during observation (MS change, worsening
headache, new or persistent vomiting), consider CT based on clinical
judgment. If child <3 months of age, consider parenteral preference and
consider scanning

refute prophylactic elevation of the head of the bed to prevent • Manage patients presenting with potential intracranial
increased intracranial pressure; while this will reduce intracra- injuries?
nial pressure, cerebral perfusion pressure will also be mildly
reduced. For treatment of increased intracranial pressure, there
is slightly more evidence for benefit of elevating the head of the ▶▶ CASE 1.8
bed (i.e., reverse Trendelenberg). “D” is incorrect since steroids
are not useful acutely in head trauma. However, steroids are A 23-year-old male is in a bar fight. He only had “two beers”
useful in cerebral edema secondary to tumor. and was just standing there “minding my own business” when
he was jumped by those infamous “two dudes” (how can
those two dudes be in so many places at once?). He presents
HELPFUL TIP:
to you about 1 hour after the event with facial trauma. His
About two-thirds of patients with a mild head injury (not
vitals are normal and he is mentating well (with the exception
deemed severe enough to obtain a CT scan) will have
of some impaired judgment secondary to the alcohol). His
some measurable decrement in function at 1 month
blood alcohol level is 150 mg/dL, showing that he is legally
secondary to post-concussion syndrome. Symptoms
intoxicated. On examination, you notice that the patient has
include headache, dizziness, difficulty concentrating,
some epistaxis and a quite swollen nose. In addition, there is
personality changes, etc. (J Emerg Med. 2011;40:262).
one avulsed tooth and one tooth that is displaced.

▶▶ Objectives: Did you learn to … Question 1.8.1 The best way to transport an avulsed
• Use the GCS? tooth is:
• Recognize which patients with head trauma are appropriate A) In sterile water
to obtain a head CT? B) In the buccal mucosa after thorough washing with soap

Ch01.indd 14 29-11-2019 12:14:54


CHAPTER 1 • Emergency Medicine 15

C) In a glass of milk B) As soon as possible to assure that there are no bone frag-
D) Wrapped in saline-soaked gauze ments threatening the brain
E) Under a pillow C) There is no need for a radiograph acutely. You can wait for 3
or 4 days
Answer 1.8.1 The correct answer is “C.” The best way to trans- D) There is never any indication for nasal radiographs
port an avulsed tooth is (1) in a glass of milk, (2) in Hanks’ bal-
anced salt solution (good luck finding this when you need it!), Answer 1.8.3 The correct answer is “C.” There is no need for
or (3) in the buccal mucosa or under the tongue in a patient in radiographs acutely except in extraordinary circumstances.
whom the risk of aspiration is not a concern. “A” is incorrect The reasons for a radiograph are to document a fracture and
because sterile water is hypotonic and may damage the tooth to assist in reduction. Because of swelling, it is difficult to get a
root decreasing the success rate of reimplantation. “B” is incor- good cosmetic result reducing a nasal fracture acutely. Thus, a
rect because washing the tooth with soap is not appropriate. radiograph is indicated in 3 to 4 days only if there is evidence
Again, you want to maintain the viability of the root if possible. of nasal deformity once swelling has resolved. If there is good
“D” is incorrect as well. If this is the only option available to cosmesis and the patient can breathe through his (they are
you, it is better than nothing, but a glass of milk or under the almost always male) nose, a radiograph is unnecessary just to
buccal mucosa is preferred. “E” is acceptable only if you are a document a fracture. “A” and “B” are incorrect because, as noted
tooth fairy. earlier, there is no reason to do a radiograph at all unless there
is evidence of deformity once the swelling is resolved. “D” is
You call the dentist who is (of course) out of town. A dentist incorrect for the reasons noted earlier.
will not be available for at least 12 hours.
Question 1.8.4 You get the epistaxis stopped and examine
Question 1.8.2 Your best course of action at this point is: the nasal mucosa. Which one of these is considered an
A) Continue to keep the tooth viable in a glass of milk emergency?
B) Continue to keep the tooth viable in the buccal mucosa A) Closed nasal fracture
C) Clean the tooth and keep it sterile and dry for reimplan- B) Septal hematoma
tation in 12 hours realizing that a bridge will probably be C) Trauma to Kiesselbach plexus
needed to hold the tooth in position D) A deviated septum
D) Reinsert the tooth into the socket yourself
Answer 1.8.4 The correct answer is “B.” A septal hematoma
Answer 1.8.2 The correct answer is “D.” If there is going to is considered an emergency. The problem is that the perichon-
be any delay in reimplantation by a dentist, the best course of drium, which supplies nutrition to the septum, is no longer in
action is to reinsert the tooth into the socket yourself. “A,” “B,” contact with the septum because of the intervening hematoma.
and “C” are all incorrect because they will reduce the rate of suc- Thus, the septal cartilage can necrose leading to a perforated
cessful reimplantation. septum. Septal hematomas should be drained acutely and the
nose packed to keep the perichondrium in contact with the
septal cartilage. “A” is incorrect (see previous question). “C”
HELPFUL TIP: is incorrect. Kiesselbach plexus is in the anterior nose and is
Primary (“baby”) teeth should NOT be re-inserted a venous plexus. Bleeding is easily controlled and generally is
into the socket! They ankylose to the bone preventing self-limited. “D,” a deviated septum, may indicate an underlying
the eruption of the permanent tooth and cause a cos- fracture but in and of itself is not an emergency.
metic deformity.
You continue to evaluate this patient and note that he has the
loss of upward gaze in the right eye, the side on which he was
HELPFUL TIP:
hit. All of the other extraocular motions are intact.
Any patient who is in the ED, says he only had three
Question 1.8.5 The most likely diagnosis in this patient is:
beers, and was “minding his own business” is probably
A) Blowout fracture with entrapment of the inferior rectus
not telling the truth on either account.
B) Blowout fracture with dysfunction of the superior rectus
C) Injury to cranial nerve III, which controls the superior AND
inferior rectus muscles
You now turn your attention to this patient’s bloody nose and D) Volitional refusal to perform upward gaze on the right side
are trying to decide whether or not to get an x-ray. in this intoxicated patient

Question 1.8.3 The BEST timing for a radiograph of the Answer 1.8.5 The correct answer is “A.” The most likely diag-
nose is: nosis is blowout fracture with entrapment of the inferior rectus.
A) As soon as possible after the trauma, once other injuries are The force of a blow to the globe is transmitted to the inferior
stabilized and more important problems are addressed orbital wall, which is the weakest point in the orbit. This can

Ch01.indd 15 29-11-2019 12:14:54


Another random document with
no related content on Scribd:
The Project Gutenberg eBook of Don
Sebastian
This ebook is for the use of anyone anywhere in the United States
and most other parts of the world at no cost and with almost no
restrictions whatsoever. You may copy it, give it away or re-use it
under the terms of the Project Gutenberg License included with this
ebook or online at www.gutenberg.org. If you are not located in the
United States, you will have to check the laws of the country where
you are located before using this eBook.

Title: Don Sebastian


or, The house of the Braganza: An historical romance. vol. 4

Author: Anna Maria Porter

Release date: December 6, 2023 [eBook #72343]

Language: English

Original publication: London: Longman, Hurst, Rees, and Orme,


1809

Credits: Chuck Greif and the Online Distributed Proofreading Team


at https://www.pgdp.net (This book was produced from
images made available by the HathiTrust Digital Library.)

*** START OF THE PROJECT GUTENBERG EBOOK DON


SEBASTIAN ***
DON SEBASTIAN;

OR,

THE HOUSE OF BRAGANZA.

J. M‘CREERY,Printer,
Black-Horse-Court, Fleet-Street, London.

DON SEBASTIAN;

OR,

THE HOUSE OF BRAGANZA.


AN HISTORICAL ROMANCE.

IN FOUR VOLUMES.

BY MISS ANNA MARIA PORTER.


AUTHOR OF THE HUNGARIAN BROTHERS.

Take Physic, Pomp!


Expose thyself to feel what wretches feel,
So shalt thou shake the superflux to them,
And shew the Heavens more just.
King Lear.

VOL. IV.

LONDON:

PRINTED FOR LONGMAN, HURST, REES, AND ORME,


PATERNOSTER ROW.
———
1809.

CHAP. I., CHAP. II., CHAP. III., CHAP. IV., CHAP. V.,
CHAP. VI., CHAP. VII., THE CONCLUSION.

DON SEBASTIAN;

OR,

THE HOUSE OF BRAGANZA.


CHAP. I.
After serious reflection Sebastian came to the determination of
sacrificing his domestic comforts to the ultimate good of his people and his
child: the conflict was over with himself, but how was he to conquer the
heart of Kara Aziek? of that tender mother, who “chid the winds of
Heaven,” if they blew too roughly on the face of her darling?—that tender
mother and faithful wife, who saw so much of virtue and happiness around
her dwelling at Cachoeira, that she knew not where the world could shew a
station more productive of either.
Dreading the excess of her grief, yet arming himself to encounter it,
Sebastian quitted his solitude, and went forth to seek her.
He found her with Blanche, just returning from their village church,
where they had been witnessing the marriage of an Indian girl with one of
the most enlightened and amiable of her tribe. The happy scene from which
they were come, had lighted up the countenances of each: Blanche was yet
too young for complete sympathy with the blushing Izamba, but her heart
sympathized with happiness of any sort; and the tear of benevolent pleasure
which stood on the cheek of her mother, called a shower over hers.
Gently walking under a long line of cedars shading the Cachoeira,
Sebastian descried them afar off.—The full and perfectly-formed figure of
Kara Aziek, her slow soft step, the gentle dignity which distinguished her,
were contrasted by the slight and budding graces of Blanche’s more airy
form. Her step was quick, bounding, and uncertain as the young Gazelle’s;
her looks were timid, not majestic; and like spring preceding summer, she
sportively advanced, admiring every object she had seen and admired a
thousand times before.
Sebastian scarcely descried her ere she was at his side: by an irresistible
impulse he took her in his arms, and holding her to his heart, suffered the
tears he could no longer restrain, to fall over her face.
It was the first time that Blanche had ever felt her father’s tears; she
looked up, and the bright roses of health and delight faded from her cheek.
“Be not alarmed, my child!” he whispered in a faltering voice, as he let
her go again, “I will rejoin your mother soon—speak not to her of this
weakness—I must explain it myself.”
Having spoken, he turned away, and hurrying towards a sugar-mill,
which he entered, as if intent on business, left Blanche to wait for her
mother, who seeing nothing extraordinary in this conduct of her husband,
entered her own habitation.
It was long ere Sebastian sufficiently recovered himself to join Kara
Aziek; the smile with which he dressed his pale countenance could not
conceal from her the unusual agitation of his heart: she fearfully inquired its
cause, and was answered by a cautious explanation of De Castro’s situation
and engagements.
Kara Aziek listened to him in profound silence, which she did not break
till some moments after he had concluded; she then turned on him her
expressive eyes; no tears were there, but they were full of that maternal
anguish she felt called upon to control.
She looked tenderly at him, as if she believed him unable to avert the
calamity with which she was threatened, and as if she considered him
equally with herself, an object of compassion. “I submit:” she said at length,
turning her eyes from her husband and fixing them on Heaven; “There are
periods in which I dare not yield to my feelings. That God who has blessed
us with our Blanche, calls her now to become an instrument for her father’s
restoration: I may not detain her.”—
At the last words, Kara Aziek closed her eyes as if she would have shut
from her husband’s sight the anguish of her soul: a general trembling seized
her, and unable to relieve herself by tears, she made an effort to smile, and
pressing his hand, leaned her face upon his shoulder.
Sebastian gently supported her. This unresisting acquiescence, this
uncomplaining grief, affected him far more than the most violent despair: in
proportion as his Aziek appeared more worthy of happiness, her different
destiny seemed more cruel.
“Dearest and best of women!” he exclaimed, “is it to day that I am to
receive the strongest proof of that love which has been the angel of my life?
—You know my heart, and you spare me the misery of contending with
tenderness for you, and duty to Portugal: you weep not, you complain not!
—O my Aziek, am I then indeed, dearer to thee than the child to whom thou
gavest birth?—I expected lamentation, remonstrance, shall I confess it?—
sorrowful reproaches—I find gentleness and heroism; I find that I am still
the first in that precious heart.”—
Transported out of himself by such a conviction, Sebastian folded his
arms around his wife, whose countenance suddenly glowing with vivid
emotion, was now bathed in tears. Instantaneously melted by this burst of
affection, she wept profusely, but her tears had no bitterness in them; she
forgot at that moment the impending evil she had been contemplating.
By degrees this rapture subsided, and the separation from her daughter
returned in all its force. “I will try to merit these kind praises:” she said
faintly, “but my heart may not always have such strength: pardon me
therefore Sebastian, if some moments of weakness should make me the
selfish creature you feared to find me. Remember that in this discourse I
have spoken my real sentiments, and do not attend to the temporary ravings
of a mother, who cannot always hear the voices of Reason and Religion;
who cannot always obey their commands. In my soul I am convinced we
ought to make this sacrifice; as such, it shall be completed.”
Again the tears of Kara Aziek ceased to flow, and her features resumed
their former paleness. Sebastian still looked at her with a mixture of anguish
and delight. His affection was eloquent, and repeated tributes to the
fortitude he admired, contributed to support and to console Kara Aziek.
One important matter yet remained to be discussed; should they or
should they not suffer Blanche to depart without knowing the story of her
birth?
Many arguments in favor of each line of conduct presented themselves
during this interesting discussion; but those had the most weight, which
dictated explanation.
Blanche was of an age and a character to feel the value of such a
confidence: the knowledge of her parents misfortunes would surely endear
them to her heart; and when widely separated, that anxiety which must
result from her acquaintance with their critical situation, would form still a
link of union. Their thoughts, their wishes, their solicitudes, would yet
remain the same, though their persons might be divided; it would be
impossible for Blanche not to remember and to love her parents, when her
dearest interests were inseparably interwoven with their images.
In addition to this consideration, Sebastian urged one equally important:
Blanche would sooner attain the qualities requisite for her future guidance
through life, by this early call upon them. Discretion, courage, attentive
observation of persons and events, careful calculation of actions, and their
consequences, would be the natural fruit of thus giving her a necessity for
all these properties.
With the prospect of one day filling a station of responsibility, seeing in
the example of her father the awful vicissitudes to which even monarchy is
exposed, and feeling, in her own person, the dependance of man on man,
she would avoid the risk of becoming intoxicated with a distinction which
presented itself under a shape so forbidding.
Her imagination, chastized by experience, and her heart disciplined by
early care, would mature, fix, and ennoble her character: if Providence
should call her to a throne, that education would enable her to fill it with
honour; if destined to pass her life in obscurity, the memory of her parents
lot, would teach her the emptiness of the world, and the rarer treasures of
that benevolence which makes joy to itself in every station.
If blind to the advantages of candour, Sebastian should permit Blanche
to depart in ignorance of her real condition, he reflected, that she must go
with either a sentiment of curiosity about the concealed motives of her
parents, or with a sentiment of disappointment at their seemingly-lukewarm
affection: continual deception must be practised on her; and bearing away
with her no quickening principle of anxiety, her filial love would soon
languish.
The remembrance of her happy home, would, from its very happiness,
only serve to excuse her to herself for ceasing to feel an animated interest in
its inhabitants; and delivering up her young mind to the charms and
novelties of a gay life, she would perhaps lose much of her goodness and all
her simplicity.
These reflections decided Sebastian, for Kara Aziek had decided at first,
from the mere impulse of feeling.
Having left his Aziek seeking additional strength at that sacred source
whence human virtue is derived, he sought Gaspar, and imparted to him the
resolution to which he had brought himself: Gaspar’s emotion was purely
joyful; he neither dreaded dangers nor difficulties, oceans, nor dungeons,
when the prospect of being useful to his King lay before him.
His sanguine nature made him certain that he should not be long
separated from Sebastian: the destruction of Spanish tyranny, and the
restoration of Portugal, were events that he concluded must follow the
interference of England; he was but leading his sweet young mistress to a
triumph, not to a struggle; he was but going to make the path broad and
open which led back to the throne of her ancestors.
“We part, Sire! it is true;” he said, “yet what is our parting?—we shall
meet again, and meet in happiness. I feel that Providence has now set a
period to your trials: this is the epoch destined for the recovery of your
former possessions.—We shall reach England—England will raise her
powerful arm, and as if by magic, the whole mass of foreign tyranny will
crumble to dust. My honoured young mistress will then be given to the
Portuguese as a pledge that their beloved sovereign yet lives, and will
condescend to reign over them: you will arrive, Sire, to find in your own
kingdom, power, adoration, and happiness!”
“Not adoration Gaspar,” said Sebastian, smiling kindly, “not adoration,
that is an impious tribute to kings, which, thank God, I never required, nay,
which I abhorred, even in my proudest day.—If I may regain the love of my
people, by convincing them that a parental tenderness for them glows in this
time-tried heart, I shall indeed rejoice that the meridian hours of my life are
not to pass away at Cachoeira. Marvellous destiny! (he added after a
thoughtful pause) shall I ever again find myself the ruler of a mighty nation
—the arbiter of their fates—the earthly God to punish and to reward?—
When I look back to the period in which I was this powerful creature, it
seems to me a long-past dream; suffering and seclusion, the only realities!”
“Not so, Sire!” gravely observed Gaspar, “you are a sovereign and a God
at Cachoeira; you are the happiest of husbands and of fathers,—and do you
say that there are no realities but suffering and seclusion?”
Sebastian fixed his eyes on him, with a look of generous approval,
“Ever, my friend!” he exclaimed; “ever watchful over my character as much
as my interest! I spoke, Gaspar, in a way too familiar with me: I spoke from
the impression of one recollection only; I remembered therefore my losses,
and forgot my possessions. Be satisfied, my soul is fully sensible of the rare
blessings I enjoy. Had I not lost my liberty and crown, I should never have
been the happy father and husband, never have opened my eyes on the light
of pure Christianity: this thought makes me consider my misfortunes as
benefits.”
Gaspar expressed his satisfaction at so ingenuous a confession of error,
and proceeded to name the time and preparations that would be required,
ere he could commence his voyage.
He knew that merchant-ships were then in the bay of St. Salvador, bound
for St. Lucar: in one of these, it was agreed he should procure a passage for
himself and his young charge, with whom he might easily proceed from St.
Lucar to Messina.
Nothing was more common than for the children of Brazilian settlers to
be sent to Europe for their education or health, and one of these motives
would certainly be attributed to Blanche’s separation from her parents,
should any persons think such an event of sufficient consequence to employ
their thoughts.
Charged with securing the cabin of the merchant-ship, and with
providing all things requisite for a tedious voyage, Gaspar hastened to St.
Salvador, leaving sadness in that house, which until now, never knew more
than the shadow of passing clouds.
The interview with his daughter was a trying hour to Sebastian: Kara
Aziek declined being present: her heart dreaded itself; and the nearer drew
the moment in which she was to make the sacrifice demanded, the more her
fears and her agitation increased.
“If I should fail at last!” she constantly repeated to herself, while striving
to strengthen her resolution by the recollection of Sebastian’s
commendations; “if I should disappoint his trust in my promise of
submission! ah God! pity me, succour me, support my feeble spirit, and
give me that added confidence in thy mercy, that added tenderness for his
feelings, which may successfully wrestle against the despair of a mother.”
Impressed with a sense of her own weakness, Kara Aziek fled from
every scene which could enervate her still further, courageous from that
very tenderness of character, with which her courage had to struggle, she
refused to herself the luxury of indulging her grief, and of participating in
that affecting interview which must increase it.
While she surrounded herself by various occupations, forcibly wresting
her mind towards the interest of others, Sebastian was unfolding to their
daughter the wonderful story of his youth.
Her amazement and sympathy may be imagined; they were in proportion
to the sensibility of her character: but Sebastian looked beyond these, and as
he slowly related the events of his life, and pointed out the lessons to be
learned from them, he watched their effect on her who might hereafter need
such beacons to guide her in the same course.
A serious joy warmed his heart, when he beheld the impression made by
the knowledge of her birth: looks of trouble and apprehension were
instantly diffused over her features: she was not ambitious therefore: to
inspire her with zeal for her own rights, it would be requisite to teach her
the benevolent purposes to which they might be directed.
No parts of her father’s narrative so absorbed the attention of Blanche, as
those which related to her mother. While Sebastian detailed the variety of
Kara Aziek’s destiny, and the transitions from pain to pleasure which they
had had mutually endured, the eyes of Blanche alternately shone with the
brightness of joy, or fell to the ground blinded with tears. It was evident,
that from the moment Kara Aziek appeared upon the scene, her affectionate
child saw in her love the best blessing of life, and ceasing to think of her
father’s loftier anxieties, was solely interested in reaching the moment
which gave her parents to each other.
Once, and only once, did her agitated heart burst through the restraint
which delicate timidity, and filial respect had imposed: it was at the
description of Sebastian’s return to Portugal, at the reception of Donna
Gonsalva, and the conversation of the two noblemen at the house of Lopez
Vernara.
Powerfully moved by these incidents, she suddenly threw herself on the
bosom of her father, exclaiming, in a voice broken by sobs, “Ah send me
not to such a dreadful world! let me not live with people who have thus
outraged my dearest father!”
Much affected by a sensibility as just as it was exquisite, Sebastian
strained her to his breast, and lifting up the scattered ringlets from her
cheek, he kissed it fondly. That roseate cheek, those eyes now closed with
grief, but lovely still from the long lashes which fringed them, the soft arms
that twined around his neck, and the beautiful tresses which fell dishevelled
over them, all these were so like his Aziek’s, that he repeated his caresses,
and abandoned to a moment of weakness, whispered, “Should I not risk all
things to go with my child?”
At this unexpected dawn of hope, Blanche raised her face, the sunshine
of happy youth was on it. “O my father;” she cried, “let us go together!—
my mother, my dear mother too—we cannot live without her.”
Sebastian had now recovered himself, and sorrowing to destroy the
illusion his own words had raised, he proceeded in a calmer tone, to explain
to her the weighty reasons which rendered it necessary for him to trust the
judgment of De Castro, and to act by his suggestion: he alone could decide
on the prudence of measures, which must be influenced by the conduct of
those potentates amongst whom he was acting.
Blanche had been early taught to yield to the voice of reason: the
moment she was convinced that what she wished was hostile to her better
interests, or blameable in itself, she ceased to form a wish on the subject.
She submitted to the commands of duty, as to the irreversible decrees of
Heaven.
This valuable habit of moral obedience, now enabled her to acquiesce in
the dreary prospect of separating from her parents, and committing herself
to a world, of which the history had made her afraid. She wept bitterly; but
she frequently repeated, that she saw her father could not in justice to
himself and his country, and in gratitude to those friends who were now
risking their lives for his sake by secret correspondence with the enemies of
Philip, do otherwise than grant them some rallying point like herself.
Sebastian contemplated her as she sat struggling against her grief,
kissing his hands and looking at him with humid smiles: Sebastian
contemplated her with many an admiring thought. He contrasted her
character and conduct with what his own had been at a much more forward
period of youth: at that period the whole globe was scarce large enough for
his tempestuous passions to rage in. Shame and compunction were on his
brow. “Blanche,” he exclaimed, “if the certainty that you possess the fullest
love, the most perfect approbation of your father, can give comfort to your
heart, be assured that you do possess them—never my child, never were
you half so dear or so estimable to me, as at this moment.”
Blanche started from her seat, and threw herself before him; Sebastian
bent towards her, and holding her against his breast in the lovely attitude in
which she had placed herself, he resumed his discourse. “Cultivate this self-
government which ennobles you thus in a parent’s eyes!—O my daughter,
self-government is dignity, is happiness, is dominion!—’tis the secret of
disarming adversity of its sting—’tis the virtue which comprehends all
others—’tis that which will entitle you to a crown in Heaven!—Had I been
like thee, my child, at this instant I should have nothing to reproach myself
with; I should have been dwelling in the house of my fathers, and I should
not have had to send my innocent Blanche into a faithless world in search
of that possession which my folly threw away.”
He stopt, and several deep sighs followed his words. Blanche kissed his
hand with repressed ardour; her timid voice trembled as she spoke. “You
have profited by your chastisements, dearest father, or how should I have
become the character you praise?—ah me! shall I always remain so? your
virtues have sprung up out of your trials; and mine perhaps may die under
them.”
Struck with the justice of this observation, and charmed with the salutary
humility which it proved, her gratified father pursued the theme she had
begun, and discoursing on principles and passions, on trials and
temptations, forgot the lapse of time.
Kara Aziek’s voice near the windows of the apartment in which they sat,
roused him to recollection, and taking Blanche by the hand, he went forth to
join her mother.
By the same impulse, Kara Aziek and her daughter pressed each other in
their arms without speaking; Sebastian approached, and drew them towards
him! he too was silent: after some moments of extreme emotion, they
recovered themselves, and rose from this sad embrace. The faces of Kara
Aziek and Blanche were bathed in tears, that of Sebastian was pale, but
more composed.
As if by tacit agreement, no one ventured to speak of the only subject
that engrossed their thoughts: their conversation was serious, and
interrupted by long pauses, but it was evidently connected with their most
interesting reflections.
Not till she was alone with her daughter, a few days previous to her
departure, had Kara Aziek courage to name the trial they were about to
encounter: at this instant fortitude forsook her, and the weeping Blanche
beheld for the first time, her gentle and hitherto patient mother, given up to
an agony of despair.
Periods like these, are not those in which human comfortings avail: the
soul must seek and find its comforter in itself. It must be habituated to
believe that all the decrees of Heaven are wise and good; then will sorrow
gradually subside, and a consolation past utterance will succeed to
distraction.
Experience had taught this most precious lesson to Kara Aziek; for often
had she had occasion to feel in her own person, and through that of her
husband, that

“We, ignorant of ourselves,


Beg often our own harms, which the wise powers
Deny us for our good: so find we profit
By losing of our prayers.”

The silent caresses and touching tears of her daughter, contributed to


console, rather than to afflict her: so sweet, so amiable, so excellent a
creature, was she not destined to ornament and to bless mankind?—The
heart of a fond mother answered in the affirmative; and like the sick person
who courageously bears the crisis of a disorder when the paroxysm is
expected to work his cure, she roused herself to support a parting which she
hoped would lead to permanent re-union.
Though flattering herself that many months would not elapse ere the
path should be open for Sebastian’s return to Portugal, she exhorted
Blanche, as if their separation were to endure for years: her admonitions
were few and impressive; she had nothing new to teach her daughter, but
what she had been hitherto teaching her every day by example rather than
precept, she now summed up, in easily-remembered maxims.
This discourse with her mother was never forgotten by Blanche.
Gaspar had provided the accommodations requisite for his young
mistress; all their arrangements were completed, and the day was fixed for
the sailing of the vessel which was to convey them to Europe. Sebastian
confided to his friend a packet for Don Emanuel De Castro, signifying the
extent to which he would engage himself with any foreign power willing to
assist him: this packet contained also a letter to Queen Elizabeth, which
Blanche was to deliver at their first meeting.
Sebastian had written with the dignity of a King, and the tenderness of a
father: by the warmth of his paternal expressions, he sought to make her
sensible that she could not hope to injure or distress his child with impunity.
He expressed himself strongly, yet with such courtly address, that although
Elizabeth must feel his meaning, and see his doubts, she could not openly
reply to them.
The hour of separation drew near; the ship was to sail on the morrow.
Having resolved to let no other objects share her heart with her parents,
at that moment in which she alone would occupy theirs, Blanche took leave
of the Indians on the preceding evening: their uncontrolled lamentations
affected her powerfully; and it was not till the night was far advanced, that
she recovered from the mournful impression.
Day-break awoke her: when she first opened her eyes at the rustling of
the breeze among the tall tops of a grove of Magnolias, when she saw their
beautiful foliage, and heard the birds singing from their branches, she felt a
sudden shock; for this was the last time in which she should see these trees,
or feel this air!
She was going far away! she was going from her parents! at this
conviction a death-like sickness oppressed her very heart; she sunk back on
her pillow, and believing the effort impossible, resigned herself to an excess
of despair. The tears which she shed in floods, gradually relieved her
feelings, and left them more obedient to her will: she repeated to herself the
injunctions and arguments of her parents, she strove to fix a short period for
the continuance of their separation, and having recourse to devotion, was
enabled to quit her chamber with a tolerably serene countenance.
It was yet early day; no one else was stirring, and Blanche rose thus
soon, that she might take a parting look at the recent grave of Barémel.
This playmate and guardian of her infant days, had died of old age, and
was buried by Sebastian himself under a cluster of Palmito’s in his garden.
No stone, with indecent resemblance to the monument of departed man,
marked the place where he laid; but a circle of plaited reeds enclosed the
little mound, and Blanche often stole there, to cover the bed of her favorite
with flowers.
She now moved lightly and timidly towards it, half-ashamed of a
sensibility it was amiable to feel. O lovely season of youth! how sweet art
thou to behold, with thy attendant graces of modesty, susceptibility, and
self-distrust!
The colours of the blooming flowers through which she passed painted
the soft complexion of Blanche; hers was the complexion that announced a
tender and intelligent heart, for it varied with every thought and every
feeling; it was now flushed with strong emotion.
She approached Barémel’s grave, and was going to throw herself upon it
with a violence of sorrow pardonable at her early age, when she was
checked by the apprehension of its being criminal.—“Yet he was so loving,
and faithful!” she exclaimed, tears trembling in her eyes, “I think it cannot
be wrong to remember him with affection.” Her innocent heart decided in
the affirmative, and sitting down near the grassy heap, she strewed it with
blossoms, and gave loose to her tears.
Her mind was quickly thronged with long-past images: different epochs
of holidays and festivals in which Barémel had constantly performed some
amusing part; passed in review before her. She remembered his joyful bark,
his supplicating whine, his watchful and loving looks, his unwearied
attendance of her from infancy to youth, over all the romantic region that
surrounded Cachoeira. “Poor Barémel! I can never see thee again!” she
said, sighing: that apostrophe conveyed to her heart a salutary exhortation
to take comfort on a far dearer subject.
She was indeed going to quit her parents, but not for ever; they were yet
in the flower of their lives, and Providence therefore permitted her to
anticipate their future re-union. Blanche was of a grateful disposition; she
loved to be happy, and far from partaking in the nature of those wretchedly
tempered spirits who seem to feast on discontent, and refuse to be
comforted; she opened her soul to admit the smallest particle of
consolation.
Her reflections now assumed a more serious cast, (for they were
employed in enumerating the mercies she yet possessed, and those she
might anticipate without presumption,) but they were no longer afflicting:
anxious to seize this moment of resignation for appearing before her
parents, she got up, and giving a long look of regret to the resting-place of
her early companion, turned homeward.
Sebastian was advancing in search of her: “I have only been to look at
poor Barémel’s grave,” she said blushing, “since you have told me all that I
owe to him, his memory has become almost sacred to me, he saved my dear
father’s life.”
Blanche added the last sentence with trepidation which shewed she was
eager to give an honorable excuse for what might have been deemed a
weakness. Sebastian pressed her hand as he led her forward: “like thy
mother in all things!” he observed, “ever diffident of thy best and most
engaging actions!”
They proceeded in silence to the apartment of Kara Aziek: she had yet
many things to say, or to repeat, and she was now gathering fortitude to
pronounce them calmly. Blanche sat down between her parents: each held
one of her hands, and frequently pressed it; but as yet, none of them spoke:
their eyes were fixed on the ground.
Why is it that the dearest connexions, when about to lose sight of each
other, avoid looking on the countenance they love, and which they will so
soon long to behold again? is it that the soul instinctively prepares for its
calamity, and tries to soften the pang by gradual abandonment of its
enjoyments? or is it that grief would be uncontrollable if those delightful
feelings were indulged which we are conscious must end with the removal
of the person beloved?
Whatever be the motive, its effect was seen in the family at Cachoeira:
they remained silent and immovable, drawing by stealth long and
interrupted sighs. They were summoning resolution to speak of parting.
At this moment Gaspar hastily entered; every eye was raised towards
him: “The ship sails an hour earlier than we expected, a messenger has just
come from St. Salvador to say so, we have not an instant to lose.”
Kara Aziek uttered a loud shriek, and clasped her daughter in her arms;
they had started up at Gaspar’s first words, and now remained clinging to
each other. Tears, sobs, broken exclamations, embraces repeated again and
again, were the witnesses of their sorrow.
Gaspar called on them to remember the fatal consequence of delay:
almost subdued himself, he yet had courage to appear barbarous, that he
might shorten the pangs of others.
Blanche heard him not: she flung herself alternately from the arms of
one parent to those of the other, and as she deluged their bosoms with her
tears, she called on them to assure her that their separation should not be
eternal.
Cold damps stood on the brow of Sebastian, for the pains of death were
in his heart, but his eyes were tearless. Kara Aziek was like one frantic; her
softness had given place to a wild and resisting despair: she clung to her
child, and no remonstrances had power to loosen her grasp!
Sebastian trembled for her reason, and that fear gave him strength to
accomplish what he knew to be indispensible. He advanced towards his
wife, forcibly, yet tenderly, unlocked her hands as they met round the waist
of Blanche, and hastily pushing his daughter towards Gaspar, exclaimed,
“Go, go my child! if you would not kill your mother. I have, I have blessed
you—I bless you again.”
Gaspar had seized the arm of Blanche, while his agitated master was
trying to detain Kara Aziek; he now led the former towards the door, and
lifting her up, ran with her from the house to the caloche which was to carry
them to St. Salvador.
On reaching the carriage, he found that she had fainted; less alarmed at
this natural effect of sorrow, than he would have been grieved by her
lamentations, he got into the vehicle, and supporting her against his
shoulder, proceeded to bathe her temples with a pungent essence he usually
wore about him.
His exertion succeeded, Blanche revived: she looked round, and seeing
herself on the road, she knew that all hope of present change was desperate:
her eyes closed again, but it was only to weep with less obviousness, and to
preserve their last look of her parents.

You might also like